Grays Flashcards

1
Q

A 1-year-old female is admitted to the hospital with a palpable mass within one of her labia majora. Radiographic examination reveals that a loop of intestine has herniated into the visibly enlarged labium majus. This condition is due to failure of the processus vaginalis to close off. From which of the following tissue layers is the processus derived? ⃣ A. Parietal peritoneum ⃣ B. Extraperitoneal tissue ⃣ C. Transversalis fascia ⃣ D. Dartos fascia ⃣ E. Internal abdominal oblique aponeurosis

A

1 A. The processus vaginalis (meaning sheathlike process) is composed of parietal peritoneum that precedes the testis as it “migrates” from a position in the upper lumbar wall to a position outside the abdomen. This process usually obliterates, leaving only a distal portion that surrounds most of the testis as the tunica vaginalis. Whereas these features are typical of development in the male, females also have a processus vaginalis that extends into the labia majus, although congenital inguinal hernias are more common in males than females. The other listed structures are not involved in congenital inguinal hernias. GAS 283; GA 140

How well did you know this?
1
Not at all
2
3
4
5
Perfectly
2
Q

A 3-year-old boy is admitted to the hospital with signs of acute renal failure. Radiologic studies reveal that the boy has bilateral masses involving both kidneys. Examination of biopsy material confi rms the diagnosis of Wilms tumor. Which of the following gene mutations is the most common in Wilms tumor? ⃣ A. The gene responsible for WT1 ⃣ B. The gene responsible for HGF ⃣ C. The gene responsible for VEGF ⃣ D. The gene responsible for GDNF ⃣ E. The gene responsible for FGF-2

A

2 A. Wilms tumor is a kidney malignancy that usually occurs in children. It has recently been shown that it can be caused by mutations in the WT1 gene, behaving according to Knudson’s two-hit model for tumor suppressor genes. GAS 355; GA 147

How well did you know this?
1
Not at all
2
3
4
5
Perfectly
3
Q

Fusion of the caudal portions of the kidneys during embryonic development is most likely to result in which of the following congenital conditions? ⃣ A. Bicornuate uterus ⃣ B. Cryptorchidism ⃣ C. Horseshoe kidney ⃣ D. Hypospadias ⃣ E. Renal agenesis

A

3 C. During development, the kidneys typically “ascend” from a position in the pelvis to a position high on the posterior abdominal wall. Although the kidneys are bilateral structures, occasionally the inferior poles of the two kidneys fuse. When this happens, the “ascent” of the fused kidneys is arrested by the fi rst midline structure they encounter, the inferior mesenteric artery. The incidence of horseshoe kidney is about 0.25% of the population. GAS 355; GA 147

How well did you know this?
1
Not at all
2
3
4
5
Perfectly
4
Q

Which of the following congenital malformations will most predictably result in oligohydramnios? ⃣ A. Anencephaly ⃣ B. Pyloric stenosis ⃣ C. Renal agenesis ⃣ D. Tracheoesophageal fi stula ⃣ E. Urethral atresia

A

C. In normal kidney development the kidneys function during the fetal period with the resulting urine contributing to the fl uid in the amniotic cavity. When the kidneys fail to develop (renal agenesis), this contribution to the fl uid is missing and decreased amniotic fl uid (oligohydramnios) results. GAS 355; GA 147

How well did you know this?
1
Not at all
2
3
4
5
Perfectly
5
Q

Failure to urinate during embryonic or fetal life usually causes respiratory diffi culties postnatally. Which of the following relationships best describes this situation? ⃣ A. Oligohydramnios linked with hypoplastic lungs ⃣ B. Polycystic kidneys linked to tracheoesophageal fi stula ⃣ C. Polyhydramnios D. ⃣ Renal agenesis linked to insuffi cient surfactant ⃣ E. Urethral obstruction linked to ectopic viscera

A

A. There is some evidence that oligohydramnios is linked to hypoplastic lungs. This is apparently not a genetic link but rather related to the importance of adequate amniotic fl uid in normal lung development. GAS 163, 355; GA 76, 147

How well did you know this?
1
Not at all
2
3
4
5
Perfectly
6
Q

A 4-year-old male child is admitted to the hospital with severe vomiting. Radiographic examination and history taking reveals that the boy suffers from an annular pancreas. Which of the following structures is most typically obstructed by this condition? ⃣ A. Pylorus of the stomach ⃣ B. First part of the duodenum ⃣ C. Second part of the duodenum ⃣ D. Third part of the duodenum ⃣ E. Jejunum

A

6 C. In normal pancreatic development a bifi d ventral pancreatic bud rotates around the dorsal side of the gut tube and fuses with the dorsal pancreatic bud. Rarely, a portion of the ventral bud rotates around the ventral side of the gut tube, resulting in an annular pancreas. The portion of the gut tube is the same where the main pancreatic duct enters the second part of the duodenum (along with the common bile duct). The incidence of annular pancreas is about 1 in 7000. GAS 322; GA 167

How well did you know this?
1
Not at all
2
3
4
5
Perfectly
7
Q

clinic. Diagnosis reveals that the intermediate portion of the processus vaginalis is not obliterated. Which of the following conditions will most likely result from this? ⃣ A. Hypospadias ⃣ B. Sterility ⃣ C. Congenital hydrocele ⃣ D. Ectopic testis ⃣ E. Epispadias

A

C. The distal portion of the processus vaginalis contributes to the tunica vaginalis that is related to the testis. If an intermediate portion of the processus vaginalis persists, it often fi lls with fl uid, creating a hydrocele. If the entire processus vaginalis persists, the patient is likely to develop a congenital inguinal hernia. GAS 260; GA 225

How well did you know this?
1
Not at all
2
3
4
5
Perfectly
8
Q

Testicles are absent from the scrotum of a 1-yearold male admitted to the pediatric clinic. The pediatrician examined the infant and palpated the testes in the inguinal canal. Which of the following terms is used to describe this condition? ⃣ A. Pseudohermaphroditism ⃣ B. True hermaphroditism ⃣ C. Cryptorchism ⃣ D. Congenital adrenal hyperplasia ⃣ E. Chordee

A

C. Cryptorchism, often called an undescended testis, is the result of incomplete migration of the gonad from the abdomen to a location in the scrotum where it is exposed to temperatures slightly lower than core body temperature. This is important for spermatogenesis and testicular function. A testis that cannot be surgically relocated into the scrotum is usu- ally removed because it would otherwise be prone to develop testicular cancer.

How well did you know this?
1
Not at all
2
3
4
5
Perfectly
9
Q

A 28-year-old woman who is 8 months pregnant goes to the outpatient clinic for her prenatal checkup. Ultrasound examination of the fetus reveals gastroschisis, with herniation of the small bowel into the amniotic cavity. Failure of proper formation of which of the following structure(s) has resulted in this condition? ⃣ A. Head fold ⃣ B. Tail fold ⃣ C. Neural folds ⃣ D. Lateral folds ⃣ E. Amnion

A

D. The lateral folds are key structures in forming the muscular portion of the anterior abdominal wall. Failure of the lateral folds can cause a minor defect, such as an umbilical hernia, or a major defect, such as gastroschisis. GAS 256, 299; GA 154

How well did you know this?
1
Not at all
2
3
4
5
Perfectly
10
Q

Rotation of the stomach during development results in movement of the left vagus nerve from its original position. Through approximately how many degrees of rotation does the nerve move, and what is its fi nal position? ⃣ A. 90 ° to become the anterior vagal trunk ⃣ B. 90 ° to become the posterior vagal trunk ⃣ C. 270 ° to become the anterior vagal trunk ⃣ D. 270 ° to become the posterior vagal trunk ⃣ E. 180 ° to become the right vagal trunk

A

10 A. Rotation of the gut tube is a major event in the development of the gastrointestinal system. Parts of the tube rotate 270 ° , but the proximal foregut, specifi cally that portion that forms the esophagus, rotates only 90 ° . Looking from below (the standard CT or MRI view), this rotation is counterclockwise. This brings the left vagus nerve onto the anterior surface of the esophagus as it passes through the thorax. GAS 256, 345; GA 190

How well did you know this?
1
Not at all
2
3
4
5
Perfectly
11
Q

A newborn baby was diagnosed with eventration of the diaphragm, wherein one half of the diaphragm ascends into the thorax during inspiration, but the other half contracts normally. What is the most likely cause of this condition? ⃣ A. Absence of a pleuropericardial fold ⃣ B. Absence of musculature in one half of the diaphragm ⃣ C. Failure of migration of the diaphragm ⃣ D. Failure of development of the septum transversum ⃣ E. Absence of a pleuroperitoneal fold

A

11 B. The diaphragm develops from several components. Initially, the septum transversum (which will become the central tendon) forms in the cervical region, gaining innervation from C3, C4, and C5. Later, myoblasts migrate in from the body wall to form the muscular part of the diaphragm, often considered to be two bilateral hemidiaphragms. These muscles are innervated by the phrenic nerves. Eventration of the diaphragm occurs when one muscular hemidiaphragm fails to develop. With positive pressure in the abdominal cavity, and low or negative pressure in the thoracic cavity, abdominal organs are pushed into the thorax. The pleuroperitoneal folds contribute to a portion of the diaphragm posteriorly. GAS 353; GA 67

How well did you know this?
1
Not at all
2
3
4
5
Perfectly
12
Q

A 2-day-old newborn male is cyanotic after attempts to swallow milk result in collection of the milk in his mouth. After 2 days he develops pneumonia. A tracheoesophageal fi stula is suspected. Which of the following structures has failed to develop properly? ⃣ A. Esophagus ⃣ B. Trachea ⃣ C. Tongue ⃣ D. Tracheoesophageal septum ⃣ E. Pharynx

A

2 D. The tracheoesophageal septum is the downgrowth that separates the ventral wall of the foregut (esophagus) from the laryngotracheal tube. The presence of a fi stula would result in passage of fl uid from the esophagus into the trachea and could cause pneumonia. If the esophagus did not develop correctly, as in esophageal atresia, it would end as a blind tube. This kind of defect, although associated with tracheoesophageal fi stula, is not the result of an opening into the trachea, and pneumonia would not result. Abnormal tracheal development can be associated with tracheoesophageal fi stula, therefore, but it is not the direct cause of it. Abnormal tongue development does not result in a tracheoesophageal fi stula. Abnormal development of the pharynx is not associated with a tracheoesophageal fi stula. GAS 168; GA 87

How well did you know this?
1
Not at all
2
3
4
5
Perfectly
13
Q

A 3-day-old male newborn has diffi culties in breathing. A CT scan of his chest and abdomen reveals the absence of the central tendon of the diaphragm. Which of the following structures failed to develop properly? ⃣ A. Pleuroperitoneal folds ⃣ B. Pleuropericardial folds ⃣ C. Septum transversum ⃣ D. Cervical myotomes ⃣ E. Dorsal mesentery of the esophagus

A

13 C. The septum transversum forms the central tendon of the diaphragm. The pleuroperitoneal folds form the posterolateral part of the diaphragm. The pleuropericardial folds separate the pericardial cavity from the pleural cavity and form the fi brous pericardium. The cervical myotomes form the musculature of the diaphragm. The dorsal part of the dorsal mesentery of the esophagus forms the crura of the diaphragm. GAS 156; GA 67

How well did you know this?
1
Not at all
2
3
4
5
Perfectly
14
Q

A 2-day-old female infant with fever is examined by the pediatric team. Imaging reveals malrotation of the small intestine without fi xation of the mesenteries. The vessels around the duodenojejunal junction are obstructed and the intestine is at risk of becoming gangrenous. Which of the following has occurred to cause the obstruction? ⃣ A. Diaphragmatic atresia ⃣ B. Subhepatic cecum ⃣ C. Midgut volvulus ⃣ D. Duplication of the intestine ⃣ E. Congenital megacolon

A

14 C. Midgut volvulus is a possible complication of malrotation of the midgut loop without fi xed mesentery. The small intestines twist around the vasculature that is providing support for them. This can result in ischemic necrosis of the intestine. Diaphragmatic atresia is not a cause of volvulus. Subhepatic cecum is due to failure of the descent of the cecal bud and results in the absence of an ascending colon. Duplication of the intestine would not cause volvulus because there would still be a fi xed mesentery and no free movement of the intestines. Congenital megacolon is due to faulty migration of neural crest cells into the wall of the colon, which causes a lack of parasympathetic postganglionic neurons. GAS 299; GA 154

How well did you know this?
1
Not at all
2
3
4
5
Perfectly
15
Q

15 A 5-day-old male infant is diagnosed with Hirschsprung disease. CT scan examination reveals an abnormally dilated colon. Which of the following is the most likely embryologic mechanism responsible for Hirschsprung disease? ⃣ A. Failure of neural crest cells to migrate into the walls of the colon ⃣ B. Incomplete separation of the cloaca ⃣ C. Failure of recanalization of the colon ⃣ D. Defective rotation of the hindgut ⃣ E. Oligohydramnios

A

15 A. Congenital megacolon (Hirschsprung disease) results from the failure of neural crest cells to migrate into the walls of the colon. Incomplete separation of the cloaca would result in anal agenesis either with or without the presence of a fi stula. The failure of recanalization of the colon results in rectal atresia, wherein both the anal canal and rectum exist but are not connected due to incomplete canalization or no recanalization. Defective rotation of the hindgut can cause volvulus or twisting of its contents. Oligohydramnios is a defi ciency of amniotic fl uid, which can cause pulmonary hypoplasia but would not cause Hirschsprung disease. GAS 311; GA 192

How well did you know this?
1
Not at all
2
3
4
5
Perfectly
16
Q

A 1-day-old infant has a mass protruding through her umbilicus. Physical examination reveals an umbilical hernia. A CT scan reveals that part of another organ is attached to the inner surface of the hernia. What portion of the gastrointestinal tract is most likely to be attached to the inner surface of the umbilical hernia? ⃣ A. Anal canal ⃣ B. Appendix ⃣ C. Cecum ⃣ D. Ileum ⃣ E. Stomach

A

D. The ileum is the best answer choice here because it is the most common site of Meckel diverticulum. This outpouching is a persistence of the vitelline duct and it can be attached to the umbilicus. The other answer choices are not correlated with the vitelline duct and therefore will not result in the condition discussed here. GAS 291; GA 155

How well did you know this?
1
Not at all
2
3
4
5
Perfectly
17
Q

A 38-year-old pregnant woman is admitted to the emergency department with severe vaginal bleeding. Ultrasound examination confi rms the initial diagnosis of ectopic pregnancy. Which of the following is the most common site of an ectopic pregnancy? ⃣ A. Uterine tubes ⃣ B. Cervix ⃣ C. Mesentery of the abdominal wall ⃣ D. Lower part of uterine body overlapping the internal cervical os ⃣ E. Fundus of the uterus

A

A. The most common site of ectopic pregnancy is in the uterine tubes. Implantation in the internal os of the cervix can result in placenta previa, but the internal os of the cervix is not the most common site. The other choices listed are not the most common sites of ectopic pregnancy. The fundus of the uterus is the normal site of implantation.

How well did you know this?
1
Not at all
2
3
4
5
Perfectly
18
Q

A 23-year-old woman is admitted with severe abdominal pain, nausea, and vomiting. History taking shows that the pain is acute and has been constant for 4 days. The pain began in the epigastric region and radiated bilaterally around the chest to just below the scapulae. Currently the pain is localized in the right hypochondrium. A CT scan examination reveals calcifi ed stones in the gallbladder. Which of the following nerves is carrying the afferent fi bers of the referred pain? ⃣ A. Greater thoracic splanchnic nerves ⃣ B. Dorsal primary rami of intercostal nerves ⃣ C. Phrenic nerves ⃣ D. Vagus nerves ⃣ E. Pelvic splanchnic nerves

A

A. The greater splanchnic nerve carries general visceral afferent fi bers from abdominal organs and can be involved in the occurrence of referred pain. The dorsal primary rami of intercostal nerves carry general somatic afferent fi bers. Pain from these fi bers would result in sharp, localized pain not dull and diffuse as occurs in referred pain. Although the phrenic nerve carries visceral afferent fi bers, it does not innervate the gallbladder. The vagus nerve carries visceral afferent fi bers that are important for visceral refl exes, but they do not transmit pain. The pelvic splanchnic nerves are parasympathetic nerves from S2 to S4 and contain visceral afferent fi bers that transmit pain from the pelvis but not from the gallbladder.

How well did you know this?
1
Not at all
2
3
4
5
Perfectly
19
Q

A 32-year-old male is admitted to the emergency department with groin pain. Examination reveals that the patient has an indirect inguinal hernia. Which of the following nerves is compressed by the herniating structure in the inguinal canal to give the patient pain? ⃣ A. Iliohypogastric ⃣ B. Lateral femoral cutaneous ⃣ C. Ilioinguinal ⃣ D. Subcostal ⃣ E. Pudendal

A

C. An indirect inguinal hernia occurs when a loop of bowel enters the spermatic cord through the deep inguinal ring (lateral to the inferior epigastric vessels). The ilioinguinal nerve runs with the spermatic cord to innervate the anterior portion of the scrotum and proximal parts of the genitals and could readily be compressed during an indirect inguinal hernia. The other nerves listed are not likely to be compressed by the hernia. The iliohypogastric nerve innervates the skin of the suprapubic region. The lateral femoral cutaneous nerve innervates the skin over the lateral thigh. The subcostal nerve innervates the band of skin superior to the iliac crest and inferior to the umbilicus. The pudendal nerve innervates the musculature and skin of the perineum. GAS 290; GA 140

How well did you know this?
1
Not at all
2
3
4
5
Perfectly
20
Q

A 54-year-old male is admitted to the emergency department with severe upper abdominal pain. Gastroscopy reveals a tumor in the antrum of the stomach. A CT scan is ordered to evaluate lymphatic drainage of the stomach. Which of the following lymph nodes is most likely to be involved in a malignancy of the stomach? ⃣ A. Celiac ⃣ B. Superior mesenteric ⃣ C. Inferior mesenteric ⃣ D. Lumbar ⃣ E. Hepatic

A

A. The celiac lymph nodes receive lymph drainage directly from the stomach before they drain into the cisterna chyli. The superior and inferior mesenteric lymph nodes receive drainage below the stomach and not from the stomach itself. The lumbar lymph nodes receive drainage from structures inferior to the stomach and not the stomach directly. Hepatic lymph nodes are associated with liver drainage and not drainage from the stomach.

How well did you know this?
1
Not at all
2
3
4
5
Perfectly
21
Q

During a scheduled laparoscopic cholecystectomy in a 47-year-old female patient, the resident accidentally clamped the hepatoduodenal ligament instead of the cystic artery. Which of the following vessels would most likely be occluded in this iatrogenic injury? ⃣ A. Superior mesenteric artery ⃣ B. Proper hepatic artery ⃣ C. Splenic artery ⃣ D. Common hepatic artery ⃣ E. Inferior vena cava

A

B. The proper hepatic artery is the only artery typically within the hepatoduodenal ligament and therefore would be occluded. This artery lies within the right anterior free margin of the omental (or epiploic) foramen (of Winslow). The superior mesenteric artery branches from the abdominal aorta inferior to the hepatoduodenal ligament. The splenic artery runs behind the stomach and is not located in the hepatoduodenal ligament. The common hepatic artery gives origin to the proper hepatic artery but does not run within the hepatoduodenal ligament. The inferior vena cava is located at the posterior margin of the omental foramen and therefore would not be clamped.

How well did you know this?
1
Not at all
2
3
4
5
Perfectly
22
Q

A 45-year-old male was admitted to the hospital with groin pain and a palpable mass just superior to the inguinal ligament. The patient was diagnosed with an inguinal hernia and a surgical repair was performed. During the operation the surgeon found a loop of intestine passing through the deep inguinal ring. Which of the following types of hernia was this? ⃣ A. Direct inguinal ⃣ B. Umbilical ⃣ C. Femoral ⃣ D. Lumbar ⃣ E. Indirect inguinal

A

E. Indirect hernias commonly result from herniation of the intestines through the deep inguinal ring. Direct hernias penetrate the anterior abdominal wall medial to the inferior epigastric vessels through the inguinal triangle (of Hesselbach) and do not penetrate the deep inguinal ring. Umbilical hernias exit through the umbilicus, not the deep inguinal ring. Femoral hernias exit through the femoral ring inferior to the inguinal ligament. Lumbar hernias can penetrate through superior (Grynfeltt) or inferior (Petit) lumbar triangles.

How well did you know this?
1
Not at all
2
3
4
5
Perfectly
23
Q

A 55-year-old man was admitted to the hospital with severe abdominal pain. Gastroscopy and CT scan examinations revealed a perforating ulcer in the posterior wall of the stomach. Where would peritonitis most likely develop initially? ⃣ A. Right subhepatic space ⃣ B. Hepatorenal space (of Morison) ⃣ C. Omental bursa (lesser sac) ⃣ D. Right subphrenic space ⃣ E. Greater sac

A

C. The omental bursa is located directly posterior to the stomach and therefore would be the most likely space to develop peritonitis initially. The right subhepatic space (also called the hepatorenal space, or pouch of Morison) is the area posterior to the liver and anterior to the right kidney. This space can potentially accumulate fl uid and may participate in peritonitis but primarily when the patient is in the supine position. The right subphrenic space lies just inferior to the diaphragm on the right side and is not likely to accumulate fl uid from a perforated stomach ulcer. Peritonitis could develop in this area only when the patient is in the supine position. Fluid from a perforated ulcer on the posterior aspect of the stomach is not likely to enter the greater sac.

How well did you know this?
1
Not at all
2
3
4
5
Perfectly
24
Q

A 58-year-old male alcoholic is admitted to the hospital after vomiting dark red blood (hematemesis). Endoscopy reveals ruptured esophageal varices, resulting from portal hypertension. Which of the following venous tributaries to the portal system anastomoses with caval veins to cause the varices? ⃣ A. Splenic ⃣ B. Left gastroomental ⃣ C. Left gastric ⃣ D. Left hepatic ⃣ E. Right gastric

A

24 C. The left gastric vein carries blood from the stomach to the portal vein. At the esophageal-gastric junction the left gastric vein (portal system) anastomoses with esophageal veins (caval system). High blood pressure in the portal system causes high pressure in this anastomosis, causing the ruptured esophageal varices. The splenic vein and its tributaries carry blood away from the spleen and do not form a caval-portal anastomosis. The left gastroomental vein accompanies the left gastroomental artery and joins the splenic vein with no direct anastomosis with caval veins. The left hepatic vein is a caval vein and empties into the inferior vena cava. The right gastric vein drains the lesser curvature of the stomach and is part of the portal system but does not have any caval anastomosis.

How well did you know this?
1
Not at all
2
3
4
5
Perfectly
25
Q

A 45-year-old male entered the emergency department with a complaint of severe abdominal pain. During physical examination it is observed that his cremasteric refl ex is absent. Which of the following nerves is responsible for the efferent limb of the cremasteric refl ex? ⃣ A. Ilioinguinal ⃣ B. Iliohypogastric ⃣ C. Genitofemoral ⃣ D. Pudendal ⃣ E. Ventral ramus of T12

A

C. The genitofemoral nerve originates from the ventral rami of L1 and L2. The femoral part supplies skin to the femoral triangle area, whereas the “genito” part in males travels with the spermatic cord and supplies the cremaster muscle and scrotal skin. The ilioinguinal nerve arises from L1 and supplies the skin over the root of the penis and upper part of the scrotum in the male. The iliohypogastric nerve arises from L1 (and possibly fi bers from T12) and supplies skin innervation over the hypogastric region and anterolateral gluteal region. The pudendal nerve provides innervation to the external genitalia for both sexes but does not innervate the cremaster muscle in males. The ventral ramus of T12 is also associated with the lower portion of the anterior abdominal wall and the iliohypogastric nerve; it does not contribute to the cremasteric refl ex.

How well did you know this?
1
Not at all
2
3
4
5
Perfectly
26
Q

The decision is made by emergency department surgeons to perform an exploratory laparotomy on a 32-year-old female with severe abdominal pain. Where would the incision most likely be made to separate the left and right rectus sheaths? ⃣ A. Midaxillary line ⃣ B. Arcuate line ⃣ C. Semilunar line ⃣ D. Tendinous intersection ⃣ E. Linea alba

A

E. The linea alba is formed by the intersection of aponeurotic tissues between the right and left rectus abdominal muscles. It contains the aponeuroses of the abdominal muscles and is located at the midline of the body. The midaxillary line is oriented vertically in a straight line inferior to the shoulder joint and axilla. The arcuate line (of Douglas) is a curved horizontal line that represents the lower edge of the posterior tendinous portion of the rectus abdominis sheath. An incision at this line will not separate the rectus abdominis sheaths. The semilunar line is represented by an imaginary vertical line below the nipples and usually parallels the lateral edge of the rectus sheath. The tendinous intersections of the rectus abdominis muscles divide the muscle into sections and are usually not well defi ned. An incision along these intersections would not divide the two rectus sheaths.

How well did you know this?
1
Not at all
2
3
4
5
Perfectly
27
Q

After a “tummy-tuck” (abdominoplasty) procedure is performed on a 45-year-old man, which of the following layers of the abdominal wall will hold the sutures? ⃣ A. Scarpa’s fascia (membranous layer) ⃣ B. Camper’s fascia (fatty layer) ⃣ C. Transversalis fascia ⃣ D. Extraperitoneal tissue ⃣ E. External abdominal oblique fascia

A

A. Scarpa’s fascia is the thick, membranous layer deep to the Camper’s adipose fascia in the anterior abdominal wall (subcutaneous). Because of the relatively thick, tough nature of connective tissue that makes up Scarpa’s fascia, this layer is typically the site to maintain sutures. Camper’s fascia is a fatty layer (subcutaneous) and tends not to hold sutures as well, due to the increased cellular content versus the connective tissue found in the Scarpa layer. Transversalis fascia is located deep to the abdominal musculature and associated aponeurosis. Extraperitoneal fascia is the deepest layer, adjacent to the parietal peritoneum of the anterior abdominal wall. The anterior wall of the rectus sheath is the layer just deep to Scarpa’s fascia and superfi cial to the rectus abdominis muscle anteriorly. The latter three layers are not considered to be superfi cial fascia.

How well did you know this?
1
Not at all
2
3
4
5
Perfectly
28
Q

A 49-year-old man presents with acute abdominal pain and jaundice. Radiographic studies reveal a tumor in the head of the pancreas. Which of the following structures is most likely being obstructed? ⃣ A. Common bile duct ⃣ B. Common hepatic duct ⃣ C. Cystic duct ⃣ D. Accessory pancreatic duct ⃣ E. Proper hepatic artery

A

A. The common bile duct is located at the head of the pancreas and receives contents from the cystic duct and hepatic duct. An obstruction at this site causes a backup of bile back through the common bile duct and hepatic duct, with resulting pain and jaundice. The common hepatic duct is located more superior to the head of the pancreas and leads into the cystic duct. The cystic duct allows bile to enter the gallbladder from the common bile duct (draining the liver) and releases bile to the common bile duct. The accessory pancreatic duct is not affected by an obstruction of the common bile duct due to a lack of any connections between the two ducts. The proper hepatic artery will not be obstructed, for it carries blood from the liver to the inferior vena cava.

How well did you know this?
1
Not at all
2
3
4
5
Perfectly
29
Q

A 44-year-old man is admitted to the emergency department with excessive vomiting and dehydration. Radiographic images demonstrate that part of the bowel is being compressed between the abdominal aorta and the superior mesenteric artery. Which of the following intestinal structures is most likely being compressed? ⃣ A. Second part of duodenum ⃣ B. Transverse colon ⃣ C. Third part of duodenum ⃣ D. First part of duodenum ⃣ E. Jejunum

A

C. The third part of the duodenum takes a path situated anterior to the abdominal aorta and inferior to the superior mesenteric artery (a major ventral branch of the abdominal aorta). Because the third part of the duodenum lies in the angle between (“sandwiched”) these two structures, constrictions of this portion of the duodenum can occur readily. The second part of the duodenum lies parallel with, and to the right of, the abdominal aorta and is not normally in close proximity to the superior mesenteric artery. The transverse colon takes a horizontal path through the anterior abdominal cavity but travels superior or anterior to the superior mesenteric artery. The fi rst part of the duodenum continues from the pylorus, fl exing to lead to the second part of the duodenum; thus, it is not located near the superior mesenteric artery or abdominal aorta. The jejunum is an extension of the small intestine after the duodenum and is further removed from the superior mesenteric artery.

How well did you know this?
1
Not at all
2
3
4
5
Perfectly
30
Q

During the surgical repair of a perforated duodenal ulcer in a 47-year-old male patient, the gastroduodenal artery is ligated. A branch of which of the following arteries will continue to supply blood to the pancreas in this patient? ⃣ A. Inferior mesenteric ⃣ B. Left gastric ⃣ C. Right gastric ⃣ D. Proper hepatic ⃣ E. Superior mesenteric

A

E. The superior mesenteric artery will supply the pancreas if the gastroduodenal artery is ligated. It arises immediately inferior to the celiac trunk from the thoracic aorta. Its fi rst branches are the anterior and posterior inferior pancreaticoduodenal arteries, which aid the superior pancreaticoduodenal arteries (which take origin from the gastroduodenal branch of the celiac trunk) in supplying the pancreas with oxygenated blood. The inferior mesenteric artery is the most inferior of the three main arterial branches supplying the gastrointestinal tract. It supplies the hindgut from the left colic fl exure to the rectum. The left gastric artery is the smallest branch of the celiac trunk and supplies the cardioesophageal junction, the inferior esophagus, and the lesser curvature of the stomach. Its anastomosis with branches from the thoracic aorta forms one of the four main portal-caval anastomoses. The right gastric artery arises from the common hepatic artery, which is a branch from the celiac trunk. It supplies the lesser curvature of the stomach and anastomoses with the left gastric artery. The proper hepatic artery arises from the common hepatic artery and ascends to supply the liver and gallbladder. It is one of three structures forming the portal triad and is found in the free edge of the hepatoduodenal ligament. GAS 333, 336; GA 169

How well did you know this?
1
Not at all
2
3
4
5
Perfectly
31
Q

A 70-year-old man is admitted to the emergency department with severe diarrhea. An arteriogram reveals 90% blockage at the origin of the inferior mesenteric artery from the aorta. Which of the following arteries would most likely provide collateral supply to the descending colon? ⃣ A. Left gastroepiploic artery ⃣ B. Middle colic artery ⃣ C. Sigmoid artery ⃣ D. Splenic artery ⃣ E. Superior rectal artery

A

B. The middle colic artery can provide collateral supply to the descending colon when the inferior mesenteric artery is blocked or ligated. It is one of the fi rst branches of the superior mesenteric artery and supplies the transverse colon. It provides collateral blood supply both to the ascending colon and descending colon by anastomosing with the right colic branch of the superior mesenteric artery and with the left colic artery, a branch from the inferior mesenteric artery. The left gastroepiploic artery, also known as the left gastroomental artery, is a branch of the splenic artery and supplies the greater curvature of the stomach along with the right gastroomental branch of the gastroduodenal artery. The sigmoid arteries are branches from the inferior mesenteric artery and supply the inferior portion of the descending colon, the sigmoid colon, and the rectum. The sigmoid arteries have no contributing branches to the foregut or midgut. The splenic artery is the largest artery arising from the celiac trunk. It supplies the spleen and the neck, body, and tail of the pancreas and also provides short gastric branches to the stomach. It supplies no structures in the midgut or hindgut. Finally, the superior rectal artery is the terminal branch of the inferior mesenteric artery and supplies only the rectum.

How well did you know this?
1
Not at all
2
3
4
5
Perfectly
32
Q

A 24-year-old woman has a dull aching pain in the umbilical region, and fl exion of the hip against resistance (psoas test) causes a sharp pain in the right lower abdominal quadrant. Which of the following structures is most likely infl amed to cause the pain? ⃣ A. Appendix ⃣ B. Bladder ⃣ C. Gallbladder ⃣ D. Pancreas ⃣ E. Uterus

A

A. The appendix is the most likely structure that is infl amed. It lies in the right lower quadrant, and of the choices provided, it is most closely associated with the umbilical region by way of referral of pain. The patient also exhibited a positive psoas sign when fl exion of the hip against resistance was attempted. This is because the iliopsoas muscle group lies directly beneath the appendix, and upon fl exion of this muscle group, contact and direct irritation to the appendix can occur. The bladder lies inferior to the umbilicus within the pelvis and is not related to the site of pain or with a positive psoas sign. The gallbladder lies inferior to the liver and is positioned in the upper right abdominal quadrant, which is superior to the umbilicus. It is not associated with a positive psoas sign. The pancreas lies behind the stomach and is positioned between the spleen and the duodenum. It therefore lies in the upper left quadrant and is superior to the umbilicus. The uterus is located within the pelvis and is positioned antefl exed and anteverted over the bladder. It lies inferior and medial to the iliopsoas group and would not be affected by fl exion of these muscles.

How well did you know this?
1
Not at all
2
3
4
5
Perfectly
33
Q

A 35-year-old male is admitted to the hospital from the emergency department because of excruciating pain in the back and left shoulder. A CT scan reveals an abscess in the upper part of the left kidney, but no abnormality is detected in the shoulder region. The shoulder pain may be caused by the spread of the infl ammation to which of the following neighboring structures? ⃣ A. Descending colon ⃣ B. Diaphragm ⃣ C. Duodenum ⃣ D. Liver ⃣ E. Pancreas

A

B. The abscess may have spread to the diaphragm and be causing the referred shoulder pain. This is because the diaphragm lies in close proximity to the inferior poles of the kidneys. The diaphragm is innervated by the phrenic nerves, bilaterally, which descend to the diaphragm from spinal nerve levels C3, C4, and C5. It is probably at the spinal cord that the referral of pain occurs between the phrenic nerve and somatic afferents entering at those levels. The descending colon is innervated by parasympathetic nerves from S2 to S4 and visceral afferents, which do not carry pain. The duodenum is innervated by the vagus nerve, which innervates the gastrointestinal tract to the left colic fl exure. The liver is innervated sympathetically from the celiac ganglion; the parasympathetic nerves to the liver are by the vagus nerve. Neither of these two sources of innervation enters the spinal cord at the level of the shoulder and therefore could not cause referred pain to the shoulder. The pancreas is innervated by the vagus nerve, branches from the celiac ganglion, and the pancreatic plexus. None of these nerves enters the spinal cord at the level of the shoulder and therefore cannot facilitate referral of pain to the shoulder.

How well did you know this?
1
Not at all
2
3
4
5
Perfectly
34
Q

A 62-year-old man is admitted to the hospital with dull, diffuse abdominal pain. A CT scan reveals a tumor at the head of the pancreas. The abdominal pain is mediated by afferent fi bers that travel initially with which of the following nerves? ⃣ A. Greater thoracic splanchnic ⃣ B. Intercostal ⃣ C. Phrenic ⃣ D. Vagus ⃣ E. Subcostal

A

A. The afferent fi bers mediating the pain from the head of the pancreas run initially with the greater thoracic splanchnic nerves. The greater splanchnic nerves arise from sympathetic ganglia at the levels of T5 to T9 and innervate structures of the foregut and thus the head of the pancreas. Running within these nerves are visceral afferent fi bers that relay pain from foregut structures to the dorsal horn of the spinal cord. Also entering the dorsal horn are the somatic afferents from that vertebral level, which mediate pain from the body wall. Intercostal nerves T1 to T12 provide the terminal part of the pathway to the spinal cord of visceral afferents for pain from the thorax and much of the abdomen. Therefore, pain fi bers from the pancreas pass by way of the splanchnic nerves to the sympathetic chains and then, by way of communicating rami, to ventral rami of intercostal nerves, fi nally entering the spinal cord by way of the dorsal roots. The phrenic nerve innervates the diaphragm and also carries visceral afferents from mediastinal pleura and the pericardium, but it does not carry with it any visceral afferent fi - bers from the pancreas. The vagus nerve innervates the pancreas with parasympathetic fi bers and ascends all the way up to the medulla where it enters the brain. It has no visceral afferent fi bers for pain. The subcostal nerve is from the level of T12 and innervates structures below the pancreas and carries no visceral afferents from the pancreas. GAS 318-319, 342; GA 191

How well did you know this?
1
Not at all
2
3
4
5
Perfectly
35
Q

A 52-year-old male with a history of smoking and hypercholesterolemia is diagnosed with severe atherosclerosis affecting the arteries of his body. Laboratory examination reveals extremely low sperm count. Which of the following arteries is most likely occluded? ⃣ A. External iliac ⃣ B. Inferior epigastric ⃣ C. Umbilical ⃣ D. Testicular ⃣ E. Deep circumfl ex iliac

A

D. The testicular artery originates from the abdominal aorta and travels with the spermatic cord, leading to the testes in the male. The external iliac artery is located “downstream” to the origin of the testicular artery from the aorta and would not cause any problems in sperm count. The inferior epigastric artery originates close to the deep inguinal ring (spermatic cord exit) as a branch of the external iliac artery and is not associated with the testicular production of sperm. The umbilical artery originates from the internal iliac artery and is divided in adults: one part is obliterated (medial umbilical artery), and the other part gives origin to superior vesical arteries to the urinary bladder. The umbilical artery plays no role in sperm production.

How well did you know this?
1
Not at all
2
3
4
5
Perfectly
36
Q

In a routine visit to the outpatient clinic for his annual checkup, a 42-year-old male is informed that radiographic examination has given strong evidence that he has a malignancy of his scrotum. Which of the following nodes are the fi rst lymph nodes that drain the affected area? ⃣ A. Superfi cial inguinal ⃣ B. Internal iliac ⃣ C. Lumbar ⃣ D. Presacral ⃣ E. Axillary

A

A. The lymph drainage of the scrotum is into the superfi cial inguinal nodes. The internal iliac lymph nodes drain the pelvis, perineum, and gluteal region. The lumbar nodes drain lymph from kidneys, the adrenal glands, testes or ovaries, uterus, and uterine tubes. They also receive lymph from the common internal or external nodes. Axillary lymph nodes drain the anterior abdominal wall above the umbilicus. GAS 494; GA 254

How well did you know this?
1
Not at all
2
3
4
5
Perfectly
37
Q

A 35-year-old male is admitted to the hospital with an indirect inguinal hernia. During an open hernioplasty (in contrast to a laparoscopic procedure), the spermatic cord and the internal abdominal oblique muscles are identifi ed. Which component of the spermatic cord is derived from the internal abdominal oblique muscle? ⃣ A. External spermatic fascia ⃣ B. Cremaster muscle ⃣ C. Tunica vaginalis ⃣ D. Internal spermatic fascia ⃣ E. Dartos fascia

A

B. The contents of the spermatic cord include ductus deferens; testicular, cremasteric, and deferential arteries; the pampiniform plexus of testicular nerve; the genital branch of the genitofemoral nerve; the cremasteric nerves; and the testicular sympathetic plexus and also lymph vessels. The cremaster muscle and fascia originate from the internal abdominal oblique muscle. The external spermatic fascia is derived from the aponeurosis and fascia of the external oblique muscle. The tunica vaginalis is a continuation of the processus vaginalis (from parietal peritoneum) that covers the anterior and lateral sides of the testes and epididymis. The internal spermatic fascia is derived from the transversalis fascia. The dartos tunic consists of a blending of the adipose (Camper) and membranous (Scarpa) layers of the superfi cial fascia, with interspersed smooth muscle fi bers.

How well did you know this?
1
Not at all
2
3
4
5
Perfectly
38
Q

A 63-year-old man with a history of alcoholism is brought to the emergency department with hematemesis (vomiting blood). Findings on endoscopic examination suggest bleeding from esophageal varices. The varices are most likely a result of the anastomoses between the left gastric vein and which other vessel or vessels? ⃣ A. Azygos system of veins ⃣ B. Inferior vena cava ⃣ C. Left umbilical vein ⃣ D. Superior mesenteric vein ⃣ E. Subcostal veins

A

A. Esophageal varices are dilated veins in the submucosa of the lower esophagus. They often result from portal hypertension due to liver cirrhosis. The left gastric vein and the esophageal veins of the azygos system form an important portal-caval anastomosis when pressure in the portal vein, and in turn the left gastric vein, is increased. None of the other choices forms important portal-caval anastomoses.

How well did you know this?
1
Not at all
2
3
4
5
Perfectly
39
Q

A 34-year-old man is undergoing an emergency appendectomy. After the appendectomy has been performed successfully, the patient undergoes an exploratory laparoscopy. Which of the following anatomic features are the most useful to distinguish the jejunum from the ileum? ⃣ A. Jejunum has thinner walls compared with the ileum. ⃣ B. Jejunum has less mesenteric fat compared with the ileum. ⃣ C. Jejunum has more numerous vascular arcades compared with the ileum. ⃣ D. Jejunum has more numerous lymphatic follicles beneath the mucosa compared with the ileum. ⃣ E. Jejunum has fewer villi compared with the ileum.

A

B. The jejunum makes up the proximal two fi fths of the small intestine. There are several ways in which the ileum and jejunum differ. During surgery the easiest way to distinguish the two based on appearance is the relative amount of mesenteric fat. The jejunum has less mesenteric fat than the ileum. Although the jejunum does have thicker walls, more villi, and higher plicae circulares compared with the ileum, these distinctions are not visible unless the intestinal wall is incised. The jejunum has fewer vascular arcades in comparison with the ileum. Lymphatic follicles are visible, usually only histologically, in the ileum.

How well did you know this?
1
Not at all
2
3
4
5
Perfectly
40
Q

After a mastectomy, a musculocutaneous fl ap is used to restore the thoracic contour in a 34-year-old female patient. The ipsilateral (same side) rectus abdominis muscle was detached carefully from the surrounding structures and transposed to the thoracic wall. Which of the following landmarks is most often used to locate the inferior end of the posterior, tendinous layer of the rectus sheath? ⃣ A. Intercristal line ⃣ B. Linea alba ⃣ C. Arcuate line ⃣ D. Pectineal line ⃣ E. Semilunar line

A

C. The arcuate line is a horizontal line that demarcates the lower limit of the posterior aponeurotic portion of the rectus sheath. It is also where the inferior epigastric vessels perforate the sheath to enter the rectus abdominis. The intercristal line is an imaginary line drawn in the horizontal plane at the upper margin of the iliac crests. The linea alba is a tendinous, median raphe running vertically between the two rectus abdominis muscles from the xiphoid process to the pubic symphysis. The pectineal line is a feature of the superior ramus of the pubic bone; it provides an origin for the pectineus muscle of the thigh and medial insertions for the abdominal obliques and transversus muscles. The semilunar line is the curved, vertical line along the lateral border of the sheath of the rectus abdominis.

How well did you know this?
1
Not at all
2
3
4
5
Perfectly
41
Q

An anteroposterior radiograph is taken of the lumbar region in a 31-year-old female patient who had been treated for tuberculous spondylitis at vertebral levels T12-L1. The patient has been asymptomatic for 10 years. Which of the following is the most likely site of the calcifi ed tuberculous abscess? ⃣ A. Body of pancreas ⃣ B. Cecum ⃣ C. Fundus of stomach ⃣ D. Psoas fascia ⃣ E. Suspensory ligament of the duodenum

A

D. The psoas muscles (covered in psoas fascia) originate from the transverse processes, intervertebral disks, and bodies of the vertebral column at levels T12 to L5. In the image, this fascia contains a calcifi ed tuberculous abscess. The pancreas is an elongated organ located across the back of the abdomen, behind the stomach. The tapering body extends horizontally and slightly upward to the left and ends near the spleen. The cecum is the blind-ending pouch of the ascending colon, lying in the right iliac fossa. The fundus of the stomach lies inferior to the apex of the heart at the level of the fi fth rib. The suspensory ligament of the duodenum is a fi bromuscular band that attaches to the right crus of the diaphragm.

How well did you know this?
1
Not at all
2
3
4
5
Perfectly
42
Q

A 45-year-old female is admitted to the hospital with symptoms of an upper bowel obstruction. Upon CT examination it is found that the third (transverse) portion of the duodenum is being compressed by a large vessel. Which of the following vessels will most likely be causing the compression? ⃣ A. Inferior mesenteric artery ⃣ B. Superior mesenteric artery ⃣ C. Inferior mesenteric vein ⃣ D. Portal vein ⃣ E. Splenic vein

A

B. The superior mesenteric artery arises from the aorta, behind the neck of the pancreas, and descends across the uncinate process of the pancreas and the third part of the duodenum before it enters the root of the mesentery behind the transverse colon. It can compress the third part of the duodenum. The inferior mesenteric artery passes to the left behind the horizontal portion of the duodenum. The inferior mesenteric vein is formed by the union of the superior rectal and sigmoid veins and it does not cross the third part of the duodenum. The portal vein is formed by the union of the splenic vein and the superior mesenteric vein posterior to the neck of the pancreas. It ascends behind the bile duct and the hepatic artery within the free margin of the hepatoduodenal ligament. The splenic vein is formed by the tributaries from the spleen and is superior to the third part of the duodenum.

How well did you know this?
1
Not at all
2
3
4
5
Perfectly
43
Q

A 61-year-old woman had been scheduled for a cholecystectomy. During the operation the scissors of the surgical resident accidentally entered the tissues immediately posterior to the epiploic (omental) foramen (its posterior boundary). The surgical fi eld was fi lled immediately by profuse bleeding. Which of the following vessels was the most likely source of bleeding? ⃣ A. Aorta ⃣ B. Inferior vena cava ⃣ C. Portal vein ⃣ D. Right renal artery ⃣ E. Superior mesenteric vein

A

B. The omental (epiploic) foramen (of Winslow) is the only natural opening between the lesser and greater sacs of the peritoneal cavity. It is bounded superiorly by the visceral peritoneum (liver capsule of Glisson) on the caudate lobe of the liver, inferiorly by the peritoneum on the fi rst part of the duodenum, anteriorly by the free edge of the hepatoduodenal ligament, and posteriorly by the parietal peritoneum covering the inferior vena cava. Therefore, the inferior vena cava would be the most likely source of bleeding. The aorta lies to the left of the inferior vena cava in the abdomen. The portal vein, right renal artery, and superior mesenteric vein are not borders of the epiploic foramen

How well did you know this?
1
Not at all
2
3
4
5
Perfectly
44
Q

A 32-year-old woman was admitted to the hospital with a complaint of pain over her umbilicus. Radiographic examination revealed acute appendicitis. The appendix was removed successfully in an emergency appendectomy. One week postoperatively the patient complained of paresthesia of the skin over the pubic region and the anterior portion of her perineum. Which of the following nerves was most likely injured during the appendectomy? ⃣ A. Genitofemoral ⃣ B. Ilioinguinal ⃣ C. Subcostal ⃣ D. Iliohypogastric ⃣ E. Spinal nerve T9

A

B. The ilioinguinal nerve, which arises from the L1 spinal nerve, innervates the skin on the medial aspect of the thigh, scrotum (or labia majora), and the mons pubis. It has been injured in this patient. The genitofemoral nerve splits into two branches: The genital branch supplies the scrotum (or labia majora) whereas the femoral branch supplies the skin of the femoral triangle. The subcostal nerve has a lateral cutaneous branch that innervates skin in the upper gluteal region, in addition to distribution over the lower part of the anterior abdominal wall. The iliohypogastric nerve innervates the skin over the iliac crest and the hypogastric region. Spinal nerve T9 supplies sensory innervation to the dermatome at the level of T9, above the level of the umbilicus.

How well did you know this?
1
Not at all
2
3
4
5
Perfectly
45
Q

Exploratory laparoscopy was performed on a 34-year-old male, following a successful emergency appendectomy. Which of the following anatomic relationships would be seen clearly, without dissection, when the surgeon exposes the beginning of the jejunum? A. ⃣ The second portion of the duodenum is related anteriorly to the hilum of the right kidney. ⃣ B. The superior mesenteric artery and vein pass posterior to the third part of the duodenum. ⃣ C. The portal vein crosses anterior to the neck of the pancreas. ⃣ D. The second part of the duodenum is crossed anteriorly by the attachment of the transverse mesocolon. ⃣ E. The third part of the duodenum is related anteriorly to the hilum of the left kidney

A

D. The second part of the duodenum is crossed anteriorly by the transverse mesocolon, a relationship that can be seen when the beginning of the jejunum is exposed by lifting the transverse colon superiorly. The posterior relationships of the second part of the duodenum and the portal vein cannot be seen without some dissection. The third part of the duodenum is not related anteriorly to the hilum of the left kidney.

How well did you know this?
1
Not at all
2
3
4
5
Perfectly
46
Q

A 30-year-old female patient complains that she has been weak and easily fatigued over the past 6 months. She has a 3-month acute history of severe hypertension that has required treatment with antihypertensive medications. She has recently gained 4.5 kg (10 lb) and currently weighs 75 kg (165 lb). Her blood pressure is 170/100 mm Hg. Purple striae are seen over the abdomen on physical examination and she possesses a “buffalo hump.” Fasting serum glucose concentration is 140 mg/dl. A CT scan of the abdomen shows a 6-cm mass immediately posterior to the inferior vena cava. Which of the following organs is the most likely origin of the mass? ⃣ A. Suprarenal (adrenal) gland ⃣ B. Appendix ⃣ C. Gallbladder ⃣ D. Ovary ⃣ E. Uterus

A

A. The right adrenal gland is a retroperitoneal organ on the superomedial aspect of the right kidney, partially posterior to the inferior vena cava. The appendix is a narrow, hollow tube that is suspended from the cecum by a small mesoappendix. The gallbladder is located at the junction of the ninth costal cartilage and the lateral border of the rectus abdominis, quite anterior to the pathologic mass. The ovaries and uterus are both inferior to the confl uence of the inferior vena cava.

How well did you know this?
1
Not at all
2
3
4
5
Perfectly
47
Q

An obese 45-year-old female patient with an elevated temperature comes to the physician’s offi ce complaining of nausea and intermittent, acute pain in the right upper quadrant of the abdomen during the past 2 days. She has a 24-hour history of jaundice. She has a history of gallstones. Which of the following structures has most likely been obstructed by a gallstone? ⃣ A. Common bile duct ⃣ B. Cystic duct ⃣ C. Left hepatic duct ⃣ D. Pancreatic duct ⃣ E. Right hepatic duct

A

A. The symptoms of yellow eyes and jaundice would be caused by reversal of fl ow of bile into the bloodstream. The common bile duct, if obstructed, allows no collateral pathway for drainage of bile from the liver or gallbladder. The cystic duct would block gallbladder drainage but allow for bile fl ow from the liver. Obstruction of either the right or left hepatic duct would still allow for drainage from the liver, as well as the gallbladder. The pancreatic duct is not involved in the path of bile fl ow from the liver to the duodenum. It drains pancreatic enzymes from the pancreas to the duodenum.

How well did you know this?
1
Not at all
2
3
4
5
Perfectly
48
Q

A 67-year-old man has severe cirrhosis of the liver. He most likely has enlarged anastomoses between which of the following pairs of veins? ⃣ A. Inferior phrenic and superior phrenic ⃣ B. Left colic and middle colic ⃣ C. Left gastric and esophageal ⃣ D. Lumbar and renal ⃣ E. Sigmoid and superior rectal

A

C. Cirrhosis of the liver would lead to inability of the portal system to accommodate blood fl ow. Blood backs up toward systemic circulation, draining to the inferior vena cava, with pooling at areas of portal-caval anastomoses. The left gastric vein (portal) meets the esophageal vein (caval) and enlarges or expands in instances of cirrhosis. The left colic and middle colic veins are both simply tributaries to the portal system, excluding this as the correct answer. The inferior phrenic and superior phrenic veins are both systemic veins and would not be affected by portal hypertension. The same can be said for the renal and lumbar veins, both components of the caval-systemic venous system. The sigmoidal and superior rectal veins are both components of the portal venous system and would not engorge due to the portal-caval bottleneck experienced in cirrhosis. (The anastomoses between the superior rectal veins and middle or inferior rectal veins can expand in portal hypertension as hemorrhoids.)

How well did you know this?
1
Not at all
2
3
4
5
Perfectly
49
Q

A 45-year-old male is admitted to the hospital with a massive hernia that passes through the inguinal triangle (of Hesselbach). Which of the following structures is used to distinguish a direct inguinal hernia from an indirect inguinal hernia? ⃣ A. Inferior epigastric vessels ⃣ B. Femoral canal ⃣ C. Inguinal ligament ⃣ D. Rectus abdominis muscle (lateral border) ⃣ E. Pectineal ligament

A

A. The key distinguishing feature of a direct inguinal hernia is that the direct hernia does not pass through the deep inguinal ring; it passes through the lower portion of the inguinal triangle (of Hesselbach). This triangle is bordered laterally by the inferior epigastric artery and vein; medially, it is bordered by the lateral edge of rectus abdominis; inferiorly, it is bordered by the iliopubic tract and inguinal ligament. An indirect hernia passes through the deep inguinal ring and into the inguinal canal. It often descends through the superfi cial ring into the scrotum or labium, a feature less common in a direct inguinal hernia. If the tip of the examiner’s little fi nger is inserted into the superfi cial ring, and the patient is asked to cough, an indirect inguinal hernia may be felt hitting the very tip of the little fi nger. A direct inguinal hernia will be felt against the side of the digit. Both types of inguinal hernias occur above the inguinal ligament, and both are present lateral to the lateral border of the rectus abdominis. The pubic symphysis, a midline joint between the two pubic bones, provides no information for distinguishing types of hernias. The femoral canal, a feature of the femoral sheath, passes beneath the inguinal ligament into the thigh, providing the pathway taken by a femoral hernia. The pectineal ligament lies behind, or deep to, the proximal end of the femoral canal.

How well did you know this?
1
Not at all
2
3
4
5
Perfectly
50
Q

A 36-year-old man was brought to the emergency department with a bullet wound to the abdomen. The bullet penetrated the anterior abdominal wall superior to the umbilicus. If the bullet passed directly posterior in the midline, which of the following structures was most likely to have been struck fi rst by the bullet? ⃣ A. Abdominal aorta ⃣ B. Transverse colon ⃣ C. Stomach ⃣ D. Gallbladder ⃣ E. Pancreas

A

B. The bullet would probably fi rst penetrate the transverse colon because it is the most superfi cial structure located slightly superior to the umbilicus. The abdominal aorta is located deep, on the left side of the vertebral column, and would not be encountered fi rst. The stomach is located more superior, to the left, and posterior to the transverse colon and would not be affected by the anterior-posterior trajectory of the bullet. The pancreas is located deep to the stomach and duodenum. The gallbladder is located superiorly in the upper right quadrant of the abdomen, largely under cover of the liver. This would exclude its possibility of being penetrated by the midline bullet.

How well did you know this?
1
Not at all
2
3
4
5
Perfectly
51
Q

A 48-year-old man has had three episodes of upper gastrointestinal bleeding from esophageal varices. He has a history of chronic alcoholism but has recently been rehabilitated. Further evaluation shows ascites and splenomegaly. Which of the following surgical venous anastomoses is most commonly used to relieve these symptoms and signs before a liver transplant is attempted? ⃣ A. Left gastric to splenic vein ⃣ B. Right gastric to left gastric vein ⃣ C. Right renal to right gonadal vein ⃣ D. Splenic to left renal vein ⃣ E. Superior mesenteric to inferior mesenteric vein

A

D. Surgical anastomoses to alleviate symptoms of portal hypertension are rooted in the premise that connection of a large portal vein to a large systemic vein allows for collateral drainage of the portal system. The splenic vein, a component of the portal venous system, and the left renal vein, a component of the caval-systemic venous system, are ideally located to allow for a low-resistance, easily performed anastomosis. Anastomosing the left gastric vein to the splenic vein, the right gastric vein to the left gastric vein, or the superior mesenteric vein to the inferior mesenteric vein would all be ineffectual because each of these veins is a component of just the portal venous system. In addition, the right renal and right gonadal veins are both tributaries of the caval system, and surgical connection would provide no benefi t.

How well did you know this?
1
Not at all
2
3
4
5
Perfectly
52
Q

A 55-year-old man is admitted to the hospital with nausea, vomiting, and hematuria. A CT scan examination reveals a neoplasm in the posterior surface of the inferior pole of the left kidney that has invaded through the renal pelvis, renal capsule, ureter, and fat. To which of the following regions will pain most likely be referred? ⃣ A. Skin of the anterior and lateral thighs and femoral triangle ⃣ B. Skin over the gluteal region, pubis, medial thigh, and scrotal areas ⃣ C. Skin over the medial, anterior, and lateral side of the thigh ⃣ D. Skin over the pubis and umbilicus ⃣ E. Skin over the pubis, umbilicus, and posterior abdominal wall muscles

A

B. Visceral pain from the kidneys and the ureter at the point of the neoplasm is mediated via T11 and T12 spinal cord levels. Therefore, pain is referred to these dermatomes leading to pain in the upper gluteal, pubic, medial thigh, scrotal, and labial areas (from subcostal and iliohypogastric nerves, in particular). In contrast, the umbilical region, the T10 dermatome, is supplied by the T10 spinal nerve, excluding it from being the correct answer. The dermatomes that supply the anterior and lateral thighs are of upper lumbar origin and would not receive pain referred from the kidneys.

How well did you know this?
1
Not at all
2
3
4
5
Perfectly
53
Q

A 30-year-old female patient has complained of weakness and fatigability over the past 6 months. She has a 3-month acute history of severe hypertension that has not responded to antihypertensive medications. Fasting serum glucose concentration is 140 mg/dl. A CT scan of the abdomen shows a 6-cm mass in the adrenal gland affecting the secretory cells of the adrenal medulla. Which of the following structures is most likely releasing products into the bloodstream to produce the hypertension and other signs? ⃣ A. Preganglionic sympathetic axons in thoracic splanchnic nerves ⃣ B. Cells of neural crest origin that migrated to the adrenal medulla ⃣ C. Preganglionic parasympathetic branches of the posterior vagal trunk ⃣ D. Postganglionic parasympathetic branches of the left or right vagus nerves ⃣ E. Postganglionic fi bers from pelvic splanchnic nerves

A

B. The mass leads to increased stimulation and secretions of the chromaffi n cells of the adrenal medulla. These cells are modifi ed postganglionic sympathetic neurons of neural crest origin, and the epinephrine (adrenaline) and norepinephrine (noradrenaline) released by these cells passes into the suprarenal (adrenal) veins. The adrenal medulla receives stimulation from preganglionic sympathetic fi bers carried by the thoracic splanchnic nerves. Parasympathetic neurons are not found in the adrenal medulla and would have no participation in the effects of the tumor. In addition, the pelvic splanchnic nerves are parasympathetic and do not travel to the adrenal medulla.

How well did you know this?
1
Not at all
2
3
4
5
Perfectly
54
Q

A 48-year-old man is admitted to the hospital with severe abdominal pain. Radiographic examination reveals a tumor in the tail of the pancreas. A diagnostic arteriogram shows that the tumor has compromised the blood supply to another organ. Which of the fol- ABDOMEN 64 lowing organs is most likely to have its blood supply compromised by this tumor? ⃣ A. Duodenum ⃣ B. Gallbladder ⃣ C. Kidney ⃣ D. Liver ⃣ E. Spleen

A

E. The splenic artery lies adjacent to the superior border of the pancreas. The organ it principally supplies is the spleen, which is located at the termination of the pancreatic tail. Blood supply to the spleen can therefore be affected in the event of a tumor in the tail of the pancreas. The duodenum receives blood from the gastroduodenal artery, located near the head of the pancreas. The gallbladder is supplied by the cystic artery, a branch of the hepatic artery and is not in contact with the pancreas. The liver is also supplied by the hepatic artery. The kidneys are supplied by the right and left renal arteries. The left renal artery lies deep and medial to the pancreatic tumor, and blood supply would proceed uninterrupted.

How well did you know this?
1
Not at all
2
3
4
5
Perfectly
55
Q

A 57-year-old man is admitted to the emergency department with left fl ank pain. Blood tests indicate hematuria and anemia. A magnetic resonance scan reveals that blood fl ow in the left renal vein is being occluded by an arterial aneurysm where the vein crosses the aorta. The aneurysm is most likely located in which of the following arteries? ⃣ A. Celiac ⃣ B. Inferior mesenteric ⃣ C. Left colic ⃣ D. Middle colic ⃣ E. Superior mesenteric

A

E. The superior mesenteric artery lies just superior and anterior to the left renal vein as the vein passes to its termination in the inferior vena cava. The celiac artery is located superiorly and would not compress the left renal vein. The inferior mesenteric artery and its left colic branch are located too inferiorly to occlude the left renal vein. The middle colic artery arises from the anterior aspect of the superior mesenteric artery inferior to the position of the left renal vein. An aneurysm of the superior mesenteric artery would therefore be most likely to occlude the left renal vein.

How well did you know this?
1
Not at all
2
3
4
5
Perfectly
56
Q

A 57-year-old man is admitted to the emergency department with pain in his left fl ank and testicles. Laboratory tests indicate hematuria and anemia. A CT scan examination provides evidence that blood fl ow in the left renal vein is being occluded where it crosses anterior to the aorta. Which of the following is the most likely cause of the testicular pain? ⃣ A. Compression of the testicular artery ⃣ B. Occlusion of fl ow of blood in the testicular vein ⃣ C. Compression of the afferent fi bers in the lumbar splanchnic nerves ⃣ D. Compression of the sympathetic fi bers in the preaortic plexus ⃣ E. Compression of the posterior vagus nerve

A

B. Blood fl ow would be impeded or greatly reduced in the left testicular vein because of the occlusion of the left renal vein—into which the left testicular vein drains. This would result in pain as the testicular venous vessels become swollen. The testicular artery originates from the abdominal aorta more inferiorly and is not being compressed. Pain mediated from the renal organs would pass to the T11 and T12 spinal cord levels via the thoracic splanchnic nerves. There would be no compression of lumbar splanchnic nerves in this case. Compression of the preaortic sympathetics would not produce pain, nor would it cause referral of pain. Visceral afferents for pain terminate at the T7 level of the spinal cord. The vagus, a parasympathetic nerve, does not carry visceral pain fi bers in the abdomen; pain is mediated by branches of the sympathetic chains.

How well did you know this?
1
Not at all
2
3
4
5
Perfectly
57
Q

A 51-year-old woman is admitted to the hospital with an acutely painful abdomen. Radiographic examination reveals penetration of the fundic region of the stomach by an ulcer, resulting in intraabdominal bleeding. Which of the following arteries is the most likely source of the bleeding? ⃣ A. Common hepatic artery ⃣ B. Inferior phrenic artery ⃣ C. Left gastroepiploic artery ⃣ D. Short gastric artery ⃣ E. Splenic artery

A

D. The most likely candidate for bleeding from the fundic region of the stomach in this case would be either the short gastric or dorsal gastric branches of the splenic artery. The short gastric arteries pass from the area of the splenic hilum to the fundus, supplying anterior and posterior branches to this part of the stomach. The dorsal gastric artery, which arises from the midportion of the splenic artery, passes to the dorsal aspect of the fundus. The main stem of the splenic artery would pass somewhat inferior to the location of the ulceration. The common hepatic artery and inferior phrenic artery are quite removed from the area of the ulcer. The left gastroepiploic artery courses along the greater curvature of the body of the stomach, distal to the fundus.

How well did you know this?
1
Not at all
2
3
4
5
Perfectly
58
Q

A 39-year-old woman is admitted to the hospital with pain radiating to her inguinal region. Radiographic and physical examination reveal a herniation. Which of the following is the most common type of hernia in a female patient? ⃣ A. Femoral hernia ⃣ B. Umbilical hernia ⃣ C. Direct inguinal hernia ⃣ D. Indirect inguinal hernia ⃣ E. Epigastric hernia

A

D. Indirect inguinal hernia is the most common groin hernia in females. Although femoral hernias occur more commonly in females than in males, the occurrence of indirect inguinal hernias in women is greater. Inguinal hernias are much more common in males than in females. Epigastric and umbilical hernias would not present with pain to the inguinal region. Direct inguinal hernias, while exhibiting equal incidence in both sexes, are not the most common female hernia.

How well did you know this?
1
Not at all
2
3
4
5
Perfectly
59
Q

Radiographic examination of a 42-year-old female reveals penetration of the duodenal bulb by an ulcer, resulting in profuse intraabdominal bleeding. Which of the following arteries is the most likely source of the bleeding? ⃣ A. Posterior superior pancreaticoduodenal ⃣ B. Superior mesenteric ⃣ C. Inferior mesenteric ⃣ D. Inferior pancreaticoduodenal ⃣ E. Right gastric

A

A. The posterior superior pancreaticoduodenal artery arises from the gastroduodenal artery and travels behind the fi rst part of the duodenum, supplying the proximal portion, with branches to the head of the pancreas. Duodenal ulcers commonly arise within the fi rst portion of the duodenum, thus making the posterior superior pancreaticoduodenal artery one of the more frequently injured vessels. The superior mesenteric artery supplies derivatives of the midgut from the distal half of the duodenum to the left colic fl exure. It lies inferior to the region of ulceration. The inferior pancreaticoduodenal artery arises from the superior mesenteric artery and supplies the distal portion of the second part of the duodenum, with anastomoses with its superior counterparts. The inferior mesenteric artery is responsible for supplying most of the hindgut derivatives, generally supplying intestine from the left colic fl exure to the superior aspect of the rectum. The right gastric artery is responsible for supplying the pyloric portion of the lesser curvature of the stomach.

How well did you know this?
1
Not at all
2
3
4
5
Perfectly
60
Q

A 23-year-old man is admitted to the hospital with a bulge in his scrotum. Physical examination reveals an indirect inguinal hernia. During the open hernia repair the internal spermatic fascia is identifi ed and refl ected to expose the ductus deferens and testicular vessels. Which of the following provides the internal spermatic fascial layer of the spermatic cord? ⃣ A. External abdominal oblique aponeurosis ⃣ B. Internal abdominal oblique aponeurosis ⃣ C. Transversus abdominis aponeurosis ⃣ D. Transversalis fascia ⃣ E. Processus vaginalis

A

D. The transversalis fascial layer is the source of the internal spermatic fascia. The walls of the spermatic cord consist of three layers: external spermatic fascia, cremaster muscle, and the internal spermatic fascia. The external spermatic fascia is an extension of the external oblique fascia and aponeurosis. The cremaster muscle is a derivative of the internal oblique abdominal muscle and its fascia. The processus vaginalis is a pouch of peritoneum that precedes the testis as it descends through the deep inguinal ring and inguinal canal in the seventh month of development. That portion of the processus that is normally retained forms the tunica vaginalis of the testis. Retention of the proximal part of the processus provides a pathway for a congenital indirect inguinal hernia. If a portion of the intermediate part of the processus remains, it can form a fl uid-fi lled hydrocele.

How well did you know this?
1
Not at all
2
3
4
5
Perfectly
61
Q

A 45-year-old woman is admitted to the emergency department with a complaint of severe abdominal pain. CT scan and MRI examinations reveal a tumor of the head of the pancreas involving the uncinate process. Which of the following vessels is most likely to be occluded? ⃣ A. Common hepatic artery ⃣ B. Cystic artery and vein ⃣ C. Superior mesenteric artery ⃣ D. Inferior mesenteric artery ⃣ E. Portal vein

A

C. The superior mesenteric artery arises from the aorta, deep to the neck of the pancreas, then crosses the uncinate process and third part of the duodenum. An uncinate tumor can cause compression of the superior mesenteric artery. The common hepatic artery arises superior to the body of the pancreas and is unlikely to be affected by a tumor in the uncinate region of the pancreas. The cystic artery and vein, supplying the gallbladder, are also superior to the pancreas. The inferior mesenteric artery arises at the level of L3, which is thus situated deep to and inferior to the head of the pancreas. The portal vein, formed by the confl uence of the superior mesenteric vein and splenic vein, passes deep to the neck of the pancreas.

How well did you know this?
1
Not at all
2
3
4
5
Perfectly
62
Q

A 35-year-old obese man is admitted to the hospital with jaundice and complaints of abdominal pain. Physical examination reveals an epigastric pain that migrates toward the patient’s right side and posterior toward the scapula. Radiographic examination reveals multiple gallstones, consistent with the patient’s jaundice and typical pains of cholecystitis. Which of the following structures is most likely obstructed by the gallstones? ⃣ A. Common bile duct ⃣ B. Cystic duct ⃣ C. Left hepatic duct ⃣ D. Pancreatic duct ⃣ E. Right hepatic duct

A

The common bile duct is occluded. The pattern of pain of cholecystitis (and other signs), combined with jaundice, indicates blockage of release of bile into the duodenum. The cystic duct joins the common hepatic duct to form the common bile duct. Bile is released from the gallbladder into the cystic duct in response to cholecystokinin. From the cystic duct, bile fl ows normally through the common bile duct and the hepatopancreatic ampulla (of Vater) to enter the descending duodenum. Patients will often present with multiple gallstones. Cholecystitis is an infl ammation of the gallbladder, most frequently in association with the presence of gallstones, and often resulting from a blocked cystic duct. Increasing concentration of bile in the gallbladder can precipitate a bout of infl ammation. Blockage of the cystic duct, with concomitant cholecystitis, is not necessarily associated with jaundice. An obstruction in the common hepatic duct and subsequently the common bile duct would thus prevent communication between the duodenum and the liver, causing obstructive jaundice. An occlusion in either the left or right hepatic duct might cause mild jaundice; however, gallstones might not be present. An occlusion in the pancreatic duct would result in neither gallstones nor jaundice but may cause pancreatitis.

How well did you know this?
1
Not at all
2
3
4
5
Perfectly
63
Q

A 36-year-old woman is admitted to the hospital for the imminent birth of her baby. The decision is made to perform an emergency cesarean section. A Pfannenstiel incision is used to reach the uterus by making a transverse incision through the external sheath of the rectus muscles, about 2 cm above the pubic bones. It follows natural folds of the skin and curves superior to the mons pubis. Which of the following nerves is most at risk when this incision is made? ⃣ A. T10 ⃣ B. T11 ⃣ C. Iliohypogastric ⃣ D. Ilioinguinal ⃣ E. Lateral femoral cutaneous

A

C. The anterior cutaneous branch of the iliohypogastric nerve is responsible for the innervation of the skin above the mons pubis. This nerve arises from the T12 and L1 spinal nerves and runs transversely around the abdominal wall and over the lowest portion of the rectus sheath. It is the fi rst cutaneous nerve situated superior to the mons pubis. Nerves from the T11 and the T12 ventral rami terminate below the umbilicus but superior to the mons pubis. The ilioinguinal nerve courses through the inguinal canal, commonly on the lateral side of the spermatic cord and is therefore typically inferior to the incision. The lateral femoral cutaneous nerve travels lateral to the psoas muscle and emerges from the abdomen about an inch medial to the anterior superior iliac spine, passing thereafter to the lateral aspect of the thigh.

How well did you know this?
1
Not at all
2
3
4
5
Perfectly
64
Q

A 37-year-old woman was admitted to the emergency department with high fever (39.5° C), nausea, and vomiting. Physical examination revealed increased abdominal pain in the paraumbilical region, rebound tenderness over McBurney’s point, and a positive psoas test. Blood tests showed marked leukocytosis. Which of the following is the most likely diagnosis? ⃣ A. Ectopic pregnancy ⃣ B. Appendicitis ⃣ C. Cholecystitis ⃣ D. Kidney stone ⃣ E. Perforation of the duodenum

A

B. Appendicitis is often characterized by acute infl ammation and is indicated with both a positive psoas test and rebound pain over McBurney’s point. McBurney’s point lies 1 inch lateral to the midpoint of an imaginary line in the right lower quadrant, joining the anterior superior iliac spine and the umbilicus. In patients with appendicitis, rebound tenderness may be felt over McBurney’s point after quick, deep compression of the left lower quadrant. An ectopic pregnancy would be associated with generalized abdominal pain instead of the localized pain felt over McBurney’s point. Cholecystitis results from an infl ammation of the gallbladder and would result in pain over the epigastric region shifting to the right hypochondriac region. Kidney stones result in referred pain to the lumbar or possibly inguinal regions. Perforation of the duodenum could result in pain to palpation of the abdomen, together with adynamic (paralytic) ileus, rigidity of the abdominal wall, and referral of pain to the shoulder.

How well did you know this?
1
Not at all
2
3
4
5
Perfectly
65
Q

A 56-year-old male is admitted to the hospital with severe abdominal pain. The patient has a history of “irritable bowel syndrome” affecting his rectum. Which of the following nerves will most likely be responsible for the transmission of pain in this case? ⃣ A. Lumbar sympathetic chains ⃣ B. Pelvic splanchnic nerves ⃣ C. Pudendal nerves ⃣ D. Sacral sympathetic chains ⃣ E. Vagus nerves

A

B. The visceral afferent innervation of the rectum is transmitted by way of the pelvic splanchnic nerves, which also provide the parasympathetic supply to this organ. The lumbar sympathetic chain receives sensory fi bers from the fundus and body of the uterus. The pudendal nerve provides origin for the inferior rectal nerve, the perineal nerve, and the dorsal nerve of the penis. The inferior rectal nerve supplies somatosensory fibers to the anal canal below the pectinate line and the perianal skin; the perineal nerve and dorsal nerve of the penis innervate structures of the urogenital region. The vagus nerve provides parasympathetic supply and afferent innervation (excluding pain) to the intestine proximal to the left colic fl exure. The lumbar and sacral sympathetic chains contribute sympathetic fi bers for innervation of smooth muscle and glands of certain pelvic viscera, but not sensory fi bers for the rectum.

How well did you know this?
1
Not at all
2
3
4
5
Perfectly
66
Q

A 42-year-old is female is admitted to the hospital due to blood in her stools. Physical examination reveals no signs of infl ammation, infection, or tumor. An endoscopic examination of the distal segment of the ileum reveals a lesion of the intestinal wall. Biopsy gives histologic evidence that the lesion contains gastric mucosa. Which of the following clinical conditions will most likely explain the symptoms and signs? ⃣ A. Internal hemorrhoids ⃣ B. External hemorrhoids ⃣ C. Diverticulosis ⃣ D. Meckel’s diverticulum ⃣ E. Borborygmi

A

D. Meckel diverticulum is a fi ngerlike projection of the ileum that is generally remembered by the “rule of 2s”: It occurs in about 2% of the population, is approximately 2 feet proximal from the ileocecal junction, is about 2 inches long, occurs 2 times as often in males as in females, may contain 2 types of ectopic tissue, and may be confused often with 2 different clinical conditions. The two types of ectopic tissue are gastric mucosa and pancreatic tissue. These, along with bleeding and pain, may give indications of peptic ulcer or appendicitis. Internal and external hemorrhoids involve the rectoanal area, not the ileum, in addition to which biopsy of hemorrhoids would not reveal the presence of gastric mucosa. Borborygmi are bowel sounds that occur with the passage of gas and bowel contents through the intestines. Diverticuloses are outpouchings of the colon and would therefore be lined with colic mucosa.

How well did you know this?
1
Not at all
2
3
4
5
Perfectly
67
Q

An 80-year-old male patient is admitted to the hospital with hypertension. His history includes a notation that he has had a poor appetite for some time. During physical examination it is observed that his blood pressure is 175/95 mm Hg and that he has a marked pulsation in his epigastric region. Which of the following diagnoses will most likely explain the symptoms and signs? ⃣ A. Hiatal hernia ⃣ B. Splenomegaly ⃣ C. Cirrhosis of the liver ⃣ D. Aortic aneurysm ⃣ E. Kidney stone

A

D. The aortic aneurysm often occurs between L3 and L4, below the bifurcation of the aorta, resulting in signifi cant increase in pressure, creating the marked abdominal pulsation. The remaining answer choices would be associated with referred pain and would not be likely to result in elevated blood pressure.

How well did you know this?
1
Not at all
2
3
4
5
Perfectly
68
Q

A 48-year-old female is admitted to the hospital with a distended abdomen. A CT scan examination provides evidence of the presence of ascites ( Fig. 3-1 ). In which of the following locations will an ultrasound machine most likely confi rm the presence of the ascitic fl uid with the patient in the supine position? ⃣ A. Subphrenic recess ⃣ B. Hepatorenal recess (pouch of Morison) ⃣ C. Rectouterine recess (pouch of Douglas) ⃣ D. Vesicouterine recess ⃣ E. Subhepatic recess

A

B. In a supine patient, fl uid accumulation will often occur in the pouch of Morison, which is the lowest space in the body in a supine position. The hepatorenal space is located behind the liver and in front of the parietal peritoneum covering the right kidney. The vesicouterine and rectouterine spaces are also potential areas of fl uid accumulation; however, fl uid accumulation in these spaces occurs when the patient is in an erect position rather than a supine position.

How well did you know this?
1
Not at all
2
3
4
5
Perfectly
69
Q

A 19-year-old male is admitted to the hospital after a violent automobile collision. An MRI examination reveals that the spinal cord has been transected at the L4 cord level. Which of the following portions of the intestine will most predictably lose parasympathetic innervation from the central nervous system? ⃣ A. Jejunum ⃣ B. Ascending colon ⃣ C. Ileum ⃣ D. Descending colon ⃣ E. Transverse colon

A

D. Descending colon. Below the left colic fl exure, innervation of the gastrointestinal tract is supplied by parasympathetic fi bers of the pelvic splanchnic nerves. The parasympathetic innervation of the midgut up to the descending colon is supplied by the vagus nerve. A hematoma occurring below L4 would affect innervation of the descending colon because the pelvic splanchnic nerves arise from spinal nerve levels S2 to S4. The jejunum, ascending colon, ileum, and transverse colon are all innervated by the vagus nerve.

How well did you know this?
1
Not at all
2
3
4
5
Perfectly
70
Q

A 55-year-old male is admitted to the hospital because of severe weight loss over the preceding 6-month period of time. Radiographic examination and other tests provide evidence that a tumor is causing portal hypertension. Laboratory studies reveal that the patient has fatty stool, malnutrition, and liver hypoxia. At which of the following locations is the tumor most likely located? ⃣ A. Right lobe of the liver ⃣ B. Left lobe of the liver ⃣ C. Porta hepatis ⃣ D. Falciform ligament ⃣ E. Hepatogastric ligament

A

C. The porta hepatis (transverse fi ssure of liver) transmits the proper hepatic artery, portal vein, common hepatic duct, autonomic nerves, and lymph vessels. A tumor in this region would be most detrimental because of its abundance of vessels and lymphatics that could lead to all of these symptoms when they are compromised functionally. A tumor in either the right or left lobes would not be as serious because it would not completely obstruct all of these vessels. The falciform ligament does not carry any vessels, so a tumor in this area would not lead to the symptoms described. The hepatogastric ligament is the bilaminar peritoneal connection between the liver and the lesser curvature of the stomach and is unrelated to the symptoms and signs here.

How well did you know this?
1
Not at all
2
3
4
5
Perfectly
71
Q

During a laparoscopic cholecystectomy on a 61-year-old male, which of the following arteries must be clamped to remove the gallbladder safely? ⃣ A. Common hepatic ⃣ B. Proper hepatic ⃣ C. Right hepatic ⃣ D. Left hepatic ⃣ E. Cystic

A

E. The cystic artery is the only artery listed that goes directly to the gallbladder. It is often a branch of the right hepatic artery and must be clamped before the gallbladder is cut free from its attachments. The common hepatic artery provides origin to the proper hepatic artery, which divides into right and left hepatic arteries supplying the liver, gallbladder, and biliary tree.

How well did you know this?
1
Not at all
2
3
4
5
Perfectly
72
Q

A 45-year-old woman is admitted to the hospital after her automobile left the highway in a rainstorm and hit a tree. She had been wearing a seat belt. On radiographic examination, it is observed that she has suffered fractures of the ninth and tenth rib on her left side and that she has intraabdominal bleeding. Physical examination reveals hypovolemic shock and progressive hypotension. Which of the following organs is most likely injured to result in these clinical signs? ⃣ A. Liver ⃣ B. Pancreas ⃣ C. Left kidney ⃣ D. Spleen ⃣ E. Ileum

A

D. The spleen is a large lymphatic organ that rests against the diaphragm and ribs 9, 10, and 11 in the left hypochondriac area. A laceration of the organ is often associated with severe blood loss and shock. Almost all of the liver is located in the right hypochondrium and epigastrium, although some protrudes into the left hypochondrium below the diaphragm. The left kidney lies retroperitoneally approximately at the level of the T11 to L3 vertebrae on the left side of the body. The ilium is the upper portion of the hip bone and contributes to the bony pelvis. Ileum is the distal portion of the small intestine and is pronounced the same as “ilium.”

73
Q

Two days after an appendectomy on a 45-year-old male patient, the patient has developed an elevated temperature (39° C), is hypotensive, and complains of abdominal pain. An exploratory laparotomy reveals large amounts of blood in the peritoneal cavity due to an injury to a vessel that occurred during the appendectomy. Which of the following vessels must be ligated to stop the bleeding? ⃣ A. Right colic artery ⃣ B. Right colic artery and superior rectal artery ⃣ C. Superior mesenteric artery ⃣ D. Ileocolic artery ⃣ E. Ileocolic artery and middle colic artery

A

D. The ileocolic artery is the only artery listed that directly supplies the appendix. The superior mesenteric artery gives origin to the ileocolic, right colic, and middle colic arteries; however, the latter two lie superior to the site of the appendix. The superior rectal artery is the terminal branch of the inferior mesenteric artery and supplies the lower portion of the sigmoid colon and superior rectum.

74
Q

A 42-year-old male is admitted to the hospital with severe hematemesis. Radiographic studies reveal hepatomegaly and esophageal varices. During physical examination it is observed that the patient is icteric (jaundiced) and dilated veins (“caput medusae”) are seen on his anterior abdominal wall. Which of the following venous structures is most likely obstructed for the development of caput medusae? ⃣ A. Portal vein ⃣ B. Inferior vena cava ⃣ C. Superior vena cava ⃣ D. Lateral thoracic vein ⃣ E. Superfi cial epigastric vein

A

A. Caput medusae (referring to the head of Medusa, whose hair was formed by snakes) is caused by severely elevated portal pressure, with venous refl ux from the liver to the periumbilical veins, by way of the usually collapsed veins in the ligamentum teres. The presence of caput medusae is usually associated with end-stage disease. Caput medusae is identifi ed by the appearance of engorged veins radiating toward the lower limbs. The portal vein is the central connection of these anastomoses. Obstruction of the inferior vena cava, superior vena cava, and lateral thoracic vein do not cause portal hypertension and would not produce these symptoms. The superfi cial epigastric vein also is not associated with the development of portal hypertension but could provide a collateral channel for venous drainage.

75
Q

A 58-year-old man was admitted to the hospital with complaints of pain in the right upper quadrant and jaundice. Ultrasound examination reveals numerous large gallstones in his gallbladder. Which of the following nerves would transmit the pain of cholecystitis? ⃣ A. The right vagus nerve, with referral to the inferior angle of the scapula ⃣ B. Afferent fi bers in spinal nerves T1 to T4 ⃣ C. Visceral afferent fi bers in the greater thoracic splanchnic nerve, with referral to the dermatomes from T6 to T8 ⃣ D. Sympathetic T10 to T12 portions of greater thoracic splanchnic nerve via celiac ganglion and celiac plexus ⃣ E. Afferent fi bers of dorsal primary rami of spinal nerves T6 to T8, with referral to the epigastric region

A

C. Cholecystitis is an infl ammation of the gallbladder due to increased concentration of bile or obstruction of the cystic duct by gallstones. Pain is ultimately felt in the right hypochondriac region, which corresponds to the T6 to T8 dermatomes. Sensory afferents from the viscera carry pain fi bers as they travel with sympathetic axons in the greater thoracic splanchnic nerves. Pain cannot be felt in the viscera and is therefore referred to the body wall. The vagus nerve carries visceral sensory fi bers from the head, neck, and trunk, but these do not include pain fi bers. Spinal nerves of T1 to T4 receive afferents for pain from thoracic viscera, including the heart, but not abdominal organs. Sympathetic neurons are autonomic motor nerves and therefore do not carry sensory information. Afferent fi bers of the dorsal primary rami of spinal nerves T6 to T8 convey sensory fi bers from the back but not from internal organs.

76
Q

A 15-year-old female is brought to the hospital with fever, nausea, and diffuse paraumbilical pain, which later becomes localized in the lower right quadrant. An appendectomy procedure is begun with an ABDOMEN 67 incision at McBurney’s point. Which of the following landmarks best describes McBurney’s point? ⃣ A. The midpoint of the inguinal ligament in line with the right nipple ⃣ B. Two thirds of the distance from the umbilicus to the anterior inferior iliac spine ⃣ C. A line that intersects the upper one third of the inguinal ligament ⃣ D. A line that intersects the lower third of the inguinal ligament, about 2 cm from the pubic tubercle ⃣ E. One third of the distance from the anterior superior iliac spine to the umbilicus

A

B. McBurney’s point usually corresponds to the location of the base of the appendix where it attaches to the cecum. It is found on the right side of the abdomen, about two thirds of the distance from the umbilicus to the anterior superior iliac spine. The inguinal ligament is localized lateral and inferior to the appendix and hence would not be used as a landmark.

77
Q

A 41-year-old woman is admitted to the hospital with upper abdominal pain. A gastroscopic examination reveals multiple small ulcerations in the body of the stomach. Which of the following nerves transmits the sensation of pain from this region? ⃣ A. Spinal nerves T5 to T12 ⃣ B. Greater thoracic splanchnic nerves ⃣ C. Lesser thoracic splanchnic nerves ⃣ D. Lumbar splanchnic nerves ⃣ E. Spinal nerves T12 to L2

A

B. The greater thoracic splanchnic nerves arise from the levels of the T5 to T9 thoracic sympathetic ganglia and are responsible for carrying general visceral afferents from upper abdominal organs and, therefore, from the body of the stomach. The pain fi - bers pass from the sympathetic chain to spinal nerves T5 to T9, thereafter to the spinal cord. Spinal nerves T1 to T5 do receive sensory afferents for pain but from thoracic organs. The vagus nerves do not carry afferents for pain. The lumbar splanchnic nerves are associated with the lower portion of the abdominopelvic area.

78
Q

A 68-year-old woman is admitted to the hospital with severe pain radiating from her lower back toward her pubic symphysis. Ultrasound examination reveals that a renal calculus (kidney stone) is partially obstructing her right ureter. At which of the following locations is the calculus most likely to lodge? ⃣ A. Major calyx ⃣ B. Minor calyx ⃣ C. Pelvic brim ⃣ D. Midportion of the ureter ⃣ E. Between the pelvic brim and the uterine cervix

A

C. The ureter is normally constricted to some degree as it crosses the pelvic brim from major to minor pelvis. The minor and major calyces are proximal to the ureter and not typical sites for obstruction by kidney stones. The midportion of the ureter is not a typical site for obstruction. The site of oblique entrance of the ureter into the urinary bladder is a common site for obstruction because it is compressed by bladder contents and the muscular wall. There are no common sites of obstruction between the pelvic brim and the uterine cervix.

79
Q

A 42-year-old female is admitted to the hospital after a traumatic landing while skydiving. Radiographic examination reveals a ruptured spleen. An emergency splenectomy is performed. Which of the following peritoneal structures must be carefully manipulated to prevent intraperitoneal bleeding? ⃣ A. Coronary ligament ⃣ B. Gastrocolic ligament ⃣ C. Splenorenal ligament ⃣ D. Phrenocolic ligament ⃣ E. Falciform ligament

A

C. The splenorenal ligament is the attachment of the spleen to the left kidney and is the only ligament that contains the major branches of the splenic artery to the spleen and greater curvature of the stomach. The coronary ligament is the peritoneal refl ection from the diaphragmatic surface of the liver onto the diaphragm that encloses the bare area of liver; it is not attached to the spleen. The falciform ligament is a peritoneal fold in contact with only the diaphragm and the liver. The gastrocolic ligament contains branches of the gastroomental vessels but should not be a factor in the splenectomy.

80
Q

A 74-year-old woman is admitted to the hospital with complaints of abdominal pain. Radiographic examination reveals diverticulosis and diverticulitis of the lower portion of the descending colon, with diffuse ulcerations. It is determined that the involved area of the bowel should be removed. If the patient’s anatomy follows the most typical patterns, which vessels and nerves will be cut during the operation? ⃣ A. Branches of the vagus nerve and middle colic artery ⃣ B. Superior mesenteric plexus and superior rectal artery ⃣ C. Branches of pelvic splanchnic nerves and left colic artery ⃣ D. Branches of vagus nerve and ileocolic artery ⃣ E. Left thoracic splanchnic nerve and inferior mesenteric artery

A

C. Pelvic splanchnic nerves and the left colic artery supply the descending colon. The vagus nerve supplies the bowel only to the left colic fl exure, and the middle colic artery supplies the transverse colon. The superior rectal artery supplies the lower sigmoid and rectum. The ileocolic artery supplies the cecum and ascending colon. The left thoracic splanchnic nerve has nothing to do with the descending colon.

81
Q

A 15-year-old boy underwent an appendectomy procedure. Two weeks postoperatively the patient complains of numbness of the skin over the pubic region and anterior portion of his genitals. Which of the following nerves was most likely iatrogenically injured during the operation? ⃣ A. Pudendal ⃣ B. Genitofemoral ⃣ C. Spinal nerve T10 ⃣ D. Subcostal ⃣ E. Ilioinguinal

A

E. The ilioinguinal nerve is a terminal branch of spinal nerve L1. It innervates the skin overlying the iliac crest; the anterior portion of the urogenital region; and the upper, inner thigh. Its pathway takes it below the typical site of McBurney’s point, but it can be injured with extension of an appendectomy incision. Spinal nerve T10, the genitofemoral nerve, and the pudendal nerve are not located in the area of the incision; what is more, the area of sensory defi cit does not correlate well with their injury. The genitofemoral nerve leaves the body wall at the superfi cial inguinal ring, well below the appendectomy incision. The pudendal nerve is both motor and sensory to the perineum. The genitofemoral nerve provides motor supply to the cremaster, sensory fi bers to the scrotum, and a femoral branch innervating only the skin over the femoral triangle. Spinal nerve T10 innervates the umbilical region. The subcostal nerve innervates the skin at the level of the costal margin and the lower portion of the abdominal wall above the pubic region.

82
Q

A 5-year-old boy is admitted to the hospital with projectile vomiting. Physical examination reveals severe dysphagia. Two days later the boy develops aspiration pneumonia. Esophagographic examination shows webs and strictures in the distal third of the thoracic esophagus. Which of the following developmental conditions will most likely explain the symptoms? ⃣ A. Incomplete recanalization of the esophagus during the eighth week ⃣ B. Tracheoesophageal fi stula ⃣ C. Esophageal atresia ⃣ D. Duodenal atresia ⃣ E. Duodenal stenosis

A

A. Esophageal stenosis results from a failure of esophageal recanalization in the eighth week of development, which may also cause esophageal atresia. Webs and strictures are found in an examination of the esophagus in cases of stenosis, but they are not noticed in cases of atresia. A tracheoesophageal fi stula is an abnormal passage between the trachea and the esophagus and is associated with esophageal atresia; therefore, webs and strictures would not be seen. Duodenal atresia and stenosis occur in the small intestine and would not cause aspiration pneumonia, and clinical manifestations would not be seen in the esophagus.

83
Q

The vomitus of a 5-day-old infant contains stomach contents and bile. The vomiting has continued for 2 days. Radiographic examinations reveal stenosis of the fourth part of the duodenum. The child cries almost constantly, appearing to be hungry all of the time, yet does not gain any weight. Which of the following developmental conditions will most likely explain the symptoms? ⃣ A. Patent bile duct ⃣ B. Duodenal stenosis ⃣ C. Hypertrophied pyloric sphincter ⃣ D. Atrophied gastric antrum ⃣ E. Tracheoesophageal fi stula

A

B. Duodenal stenosis is caused by incomplete recanalization of the duodenum. The vomit contains bile in addition to the stomach contents because of the location of the occlusion, distal to the ampulla of Vater (hepatopancreatic ampulla), where the common bile duct enters the small intestine. Lack of weight gain is due to constant vomiting. A patent bile duct would not cause vomiting with bile. A hypertrophied pyloric sphincter would cause projectile vomiting without the presence of bile. An atrophied gastric antrum is caused by the removal of the membranous lining of the stomach and occurs proximal to the site of the entrance of the bile duct; therefore, the vomit would not contain bile. A tracheoesophageal fi stula would not cause vomiting of stomach contents and bile because it is a defect of the respiratory system and occurs proximal to the site at which bile is added to stomach contents.

84
Q

A 4-day-old male infant vomits the contents of his stomach, but the vomitus does not appear to contain bile. The baby is obviously distressed and makes sucking movements of his lips in response to offerings to suckle by his mother or of the bottle, but he is failing to thrive. Which of the following conditions will best explain the symptoms? ⃣ A. Duodenal stenosis ⃣ B. Duodenal atresia ⃣ C. Hypertrophied pyloric sphincter ⃣ D. Atrophied gastric fundus ⃣ E. Tracheoesophageal fi stula

A

C. With hypertrophy of the pyloric sphincter and the associated narrowing of the opening, there is projectile vomiting of stomach contents, but without bile, because bile enters the duodenum distal to the pyloric constriction. Duodenal atresia, like duodenal stenosis, causes vomiting of stomach contents and bile. Vomiting begins soon after birth in cases of atresia; vomiting due to stenosis does not begin necessarily immediately after birth and can occur days after delivery. Lack of weight gain is due to constant vomiting. An atrophied gastric fundus would not produce the signs seen here. A tracheoesophageal fi stula would not cause vomiting of stomach contents and bile because it is a defect of the respiratory system and occurs proximal to the site at which bile is added to intestinal contents.

85
Q

A 5-day-old female infant has emesis (vomit) containing stomach contents and bile. The vomiting continues for 2 days. Radiographic examinations reveal stenosis of the third part of the duodenum. The child cries consistently and is constantly hungry, but she does not gain any weight. Which of the following conditions will most likely explain her symptoms? ⃣ A. Incomplete recanalization of the esophagus during the eighth week ⃣ B. Incomplete recanalization of the duodenum ⃣ C. Esophageal atresia ⃣ D. Duodenal atresia ⃣ E. Tracheoesophageal fi stula

A

B. Incomplete recanalization of the duodenum is caused either by duodenal stenosis or partial occlusion of the lumen of the duodenum and usually occurs in the distal third portion of the duodenum. This occlusion often results in vomiting of stomach contents plus bile later in life and is the reason the child was constantly hungry but did not gain weight. Incomplete recanalization of the esophagus during the eighth week of development causes esophageal stenosis and presents as webs and strictures. Esophageal atresia is generally seen with a tracheoesophageal fi stula because it is caused by the tracheoesophageal septum deviating in the posterior direction. In some cases, it may result from a failure of recanalization during the eighth week of development and presents as a fetus with polyhydramnios due to an inability to swallow amniotic fl uid. Duodenal atresia is the result of a failed reformation of the lumen of the duodenum and is associated with vomiting within the fi rst few days of birth, polyhydramnios, and the “double bubble” sign. Tracheoesophageal fi stula is an abnormal passage between the trachea and esophagus and would not be a cause for vomiting because it is associated with the respiratory system and also occurs proximal to the site of the defect.

86
Q

A 2-hour-old male infant had been diagnosed in utero with polyhydramnios. Now he is vomiting stomach contents and bile. The vomiting continues for 2 days. Radiographic examination reveals a “double bubble” sign on ultrasound scan. The child cries consistently and is constantly hungry but has lost 300 g in weight. Which of the following conditions will most likely explain the symptoms? ⃣ A. Duodenal stenosis ⃣ B. Duodenal atresia ⃣ C. Hypertrophied pyloric sphincter ⃣ D. Atrophied gastric antrum ⃣ E. Tracheoesophageal fi stula

A

B. Duodenal atresia is the result of a failed reformation of the lumen of the duodenum and is associated with vomiting within the fi rst few days of birth. Polyhydramnios is seen due to abnormal absorption of amniotic fl uid by the intestines. Finally, radiographic or ultrasound examination would review the “double bubble” sign because of distended, gas-fi lled stomach. Duodenal stenosis is caused by incomplete recanalization of the duodenum and often results in vomiting of stomach contents plus bile later in life. Hypertrophied pyloric sphincter would cause projectile vomiting. An atrophied gastric antrum is caused by the removal of the membranous lining of the stomach and occurs proximal to the site of the entrance of the common bile duct; therefore, vomit would not contain bile. Tracheoesophageal fi stula is an abnormal passage between the trachea and esophagus and would not be a cause for any of symptoms cited in the question.

87
Q

A 4-year-old male child is admitted to the hospital with severe vomiting. Radiographic examination and history taking reveal that the boy suffers from an annular pancreas. Which of the following conditions will most typically explain the symptoms? ⃣ A. The main pancreatic duct persisted as an accessory duct that opened at the minor papilla. ⃣ B. Bile ducts failed to canalize. ⃣ C. The bifi d ventral bud fused with the dorsal bud. ⃣ D. Only the dorsal pancreatic bud formed a ring of pancreatic tissue. ⃣ E. Dorsal pancreatic bud developed around the third part of the duodenum.

A

C. Annular pancreas causes duodenal obstruction due to the thick band of pancreatic tissue that surrounds and constricts the second part of the duodenum. This obstruction can be found shortly after birth or much later. Annular pancreas can result from the bifi d ventral pancreatic bud wrapping around the duodenum during development and fusing with the dorsal pancreatic bud, thereafter forming a ring. Both the dorsal and bifi d ventral pancreatic buds are involved in this process; therefore, answers A, D, and E cannot be correct because they refer to either the dorsal or ventral buds, not both. This anomaly is not involved with lack of canalization of the bile ducts.

88
Q

The surgeon decided that a 35-year-old male patient must undergo an emergency appendectomy due to rupture of his appendix. A midline incision was made for greater access to the peritoneal cavity. The surgeon noted a 5-cm-long fi ngerlike pouch on the anterior border of the ileum about 60 cm away from the ileocecal junction. Such a pouch is a remnant of which of the following developmental structures? ⃣ A. Omphaloenteric duct (yolk stalk) ⃣ B. Branch of superior mesenteric artery ⃣ C. Umbilical vesicle (yolk sac) ⃣ D. Cecal diverticulum ⃣ E. Umbilical cord ⃣ F. Urachus

A

A. A remnant of the omphaloenteric duct generally presents as an ileal (Meckel) diverticulum in the proximal portion of the omphaloenteric duct. It normally arises as a fi ngerlike pouch about 3 to 6 cm long from the antimesenteric border of the ileum and 40 to 50 cm from the ileocecal junction. The umbilical vesicle normally turns into a pear-shaped remnant about 5 mm in diameter by week 20. The cecal diverticulum is the primordium of the cecum and appendix. The Meckel diverticulum is not a remnant of the umbilical cord.

89
Q

A 3-month-old boy is admitted to the hospital with an abnormal mass of tissue protruding from his abdomen. An MRI examination reveals that the mass contains some greater omentum and some small intestine. The abnormal mass protrudes when the infant cries, strains, and coughs. Which of the following conditions will most likely explain the symptoms? ⃣ A. Umbilical hernia ⃣ B. Omphalocele ⃣ C. Gastroschisis ⃣ D. Epigastric hernia ⃣ E. Indirect inguinal hernia

A

A. An umbilical hernia results when the body wall does not close appropriately at the site of attachment of the umbilical cord. In such cases, part of the greater omentum and small intestine can herniate from the abdomen. Umbilical herniation differs from omphalocele. In congenital omphalocele, there is a failure of intestine to return to the abdominal cavity so that there is an apparent herniation of abdominal viscera into the proximal portion of the umbilical cord, without a covering of the hernia by skin. In umbilical hernia, the herniating structures are covered by subcutaneous tissue and skin. Gastroschisis is incomplete closure of the lateral folds, resulting in an epigastric hernia, in which the viscera protrude into the amniotic cavity, surrounded by amniotic fl uid. An epigastric hernia occurs through a defect in the linea alba superior to the level of the umbilicus and occurs far more commonly in adults. An indirect inguinal hernia is when the communication between the tunica vaginalis and the peritoneal cavity do not close, and a loop of intestine or a portion of another organ such as the cecum herniates through the deep inguinal ring into the inguinal canal, with possible further descent through the superfi cial inguinal ring into the scrotum or labium majus.

90
Q

Ultrasound examinations of a male fetus in the seventh month of gestation indicate a defect on his right side, lateral to the median plane, in which the viscera protrude into the amniotic cavity. Which of the following conditions will most likely explain these fi ndings? ⃣ A. Nonrotation of the midgut ⃣ B. Patent urachus ⃣ C. Abdominal contents have not returned from the umbilical cord ⃣ D. Incomplete closure of the lateral folds ⃣ E. Persistent cloacal membrane

A

D. Gastroschisis results from an incomplete closure of the lateral folds during the fourth week of development from a defect in the medial plane of the abdominal wall. This results in an epigastric hernia, with viscera that protrude into the amniotic cavity without a peritoneal covering into the amniotic cavity and are covered by amniotic fl uid. Nonrotation of the midgut results in the lower portion of the loop returning to the abdomen fi rst, the small intestine passing to the right side of the abdomen, and the large intestine lying entirely on the left. Most cases are asymptomatic; however, obstruction of the superior mesenteric artery can cause infarction and gangrene of the part of the intestine it supplies. An umbilical hernia results when an abdominal organ herniates through an umbilical ring that does not close perfectly. Such a hernia often contains part of the greater omentum and small intestine. The cloacal membrane usually ruptures during the eighth week of development, creating a communication between the anal canal and the amniotic cavity.

91
Q

A 2-hour-old male infant vomits stomach contents and bile. The vomiting continues for 2 days. In addition, abdominal distension is noted, and he is unable to pass meconium (the earliest feces to be eliminated after birth). Which of the following is the most common cause of this condition? ⃣ A. Infarction of fetal bowel due to volvulus ⃣ B. Incomplete closure of the lateral folds ⃣ C. Failure of recanalization of the ileum ⃣ D. Remnant of the proximal portion of the omphaloenteric duct ⃣ E. Nonrotation of the midgut

A

A. The inability to pass meconium denotes an obstruction of the fetal bowel. Midgut volvulus results in the twisting of the intestines and ultimately obstruction of the small and/or large intestines. Gastroschisis results from an incomplete closure of the lateral folds during the fourth week of development from a defect in the medial plane of the abdominal wall. This results in an epigastric hernia, with viscera that protrude into the amniotic cavity and are covered by amniotic fl uid. Failure of recanalization of the ileum is seen with 50% of obstructive lesions of the intestine. This obstruction is caused by stenosis or atresia of the intestines. A remnant of the omphaloenteric duct generally presents as an ileal (Meckel) diverticulum in the proximal portion of the omphaloenteric duct. It normally arises as a fi ngerlike pouch about 3 to 6 cm long from the antimesenteric border of the ileum and 40 to 50 cm from the ileocecal junction. Nonrotation of the midgut results in the lower portion of the loop returning to the abdomen fi rst, the small intestine sitting on the right side of the abdomen, and the large intestine lying entirely on the left. Most cases are asymptomatic; however, obstruction of the superior mesenteric artery can cause infarction and gangrene of the part of the intestine it supplies.

92
Q

A 5-year-old male infant is admitted to the hospital with signs of appendicitis. An operation is performed and an ileal (Meckel) diverticulum is discovered. Which of the following is the most common cause of this condition? ⃣ A. Infarction of fetal bowel due to volvulus ⃣ B. Incomplete closure of the lateral folds ⃣ C. Failure of recanalization of the ileum ⃣ D. Remnant of the proximal portion of the omphaloenteric duct ⃣ E. Nonrotation of the midgut

A

D. A remnant of the proximal portion of the omphaloenteric duct generally presents as an ileal (Meckel) diverticulum. It normally arises as a fi ngerlike pouch about 3 to 6 cm long from the antimesenteric border of the ileum and 40 to 50 cm from the ileocecal junction. Midgut volvulus results from a twisting of the intestines and, ultimately, obstruction of the small and large intestines. Infarction of fetal bowel is seen and would not be the cause of the signs of appendicitis. Gastroschisis results from an incomplete closure of the lateral folds during the fourth week of development from a defect in the medial plane of the abdominal wall. This results in an epigastric hernia, with viscera that protrude into the amniotic cavity and are covered by amniotic fl uid. Failure of recanalization of the ileum is seen in 50% of obstructive lesions of the intestine. This obstruction is caused by stenosis or atresia of the intestines. Nonrotation of the midgut results in the lower portion of the loop returning to the abdomen fi rst so that the small intestine becomes fi xed on the right side of the abdomen, with the large intestine lying entirely on the left. Most cases are asymptomatic; however, obstruction of the superior mesenteric artery can cause infarction and gangrene of the part of the intestine it supplies.

93
Q

A newborn male infant has no passage of fi rst stool (meconium) for 48 hours after birth. Physical examination reveals that the young patient has anal agenesis with a perineal fi stula. Which of the following is the most common cause of this condition? ⃣ A. Incomplete separation of the cloaca by the urorectal septum ⃣ B. Dorsal deviation of the urorectal septum ⃣ C. Failure of the anal membrane to perforate ⃣ D. Abnormal recanalization of the colon ⃣ E. Remnant of the proximal portion of the omphaloenteric duct

A

A. Incomplete separation of the cloaca by the urorectal septum results in anal agenesis. Dorsal deviation of the urorectal septum would result in anal stenosis. Failure of the anal membrane to perforate results in imperforate anus, not anal agenesis. Abnormal recanalization of the colon results in rectal atresia, in which there is no connection between the rectum and anal canal. Remnants of the proximal portion of the omphaloenteric duct would result in a Meckel diverticulum, not anal agenesis.

94
Q

A 3-month-old female infant is diagnosed with anal stenosis after several periods of stool infrequency, two of which lasted 10 days without a bowel movement. Which of the following is the most likely cause of this condition? ⃣ A. Incomplete separation of the cloaca by the urorectal septum ⃣ B. Dorsal deviation of the urorectal septum ⃣ C. Failure of the anal membrane to perforate ⃣ D. Abnormal recanalization of the colon ⃣ E. Remnant of the proximal portion of the omphaloenteric duct

A

B. Dorsal deviation of the urorectal septum results in anal stenosis. Incomplete separation of the cloaca by the urorectal septum results in anal agenesis. Failure of the anal membrane to perforate results in imperforate anus. The anal canal exists but is obstructed by a layer of tissue. Abnormal recanalization of the colon results in rectal atresia, in which there is no connection between the rectum and anal canal. Remnants of the proximal portion of the omphaloenteric duct would result in a Meckel diverticulum, not anal stenosis.

95
Q

A 2-month-old male infant presents with a fecal discharge from his umbilicus. Which of the following diagnoses will best explain this condition? ⃣ A. Enterocystoma ⃣ B. Vitelline cyst ⃣ C. Ileal (Meckel) diverticulum ⃣ D. Vitelline fi stula ⃣ E. Volvulus

A

D. A vitelline fi stula is caused by the persistence of the vitelline duct, which can, by its connections with the umbilicus, cause the symptoms described. An enterocystoma is a tumor and would not result in the symptoms described. A vitelline cyst is a persistence of the vitelline duct; however, it would not open directly to the outside so would not cause the symptoms. Meckel diverticulum can result in a vitelline fi stula, but it is simply the persistence of the vitelline duct that can appear in different forms (such as cyst or fi stula). Volvulus is the twisting of the small intestines around their suspending vasculature. It can result from malrotation of the midgut loop and would not result in the symptoms described.

96
Q

A 5-day-old male infant is diagnosed with anorectal agenesis. An ultrasound study reveals a rectourethral fi stula. Which of the following is the most likely embryologic cause of this condition? ⃣ A. Failure of the proctodeum to develop ⃣ B. Agenesis of the urorectal septum ⃣ C. Failure of fi xation of the hindgut ⃣ D. Abnormal partitioning of the cloaca ⃣ E. Premature rupture of the anal membrane

A

D. Anorectal agenesis is due to abnormal partitioning of the cloaca and is often associated with a rectourethral, rectovaginal, or rectouterine fi stula. Failure of fi xation of the hindgut can result in volvulus. Failure of the proctodeum to develop will result in an imperforate anus. Agenesis of the urorectal septum would most likely lead to a fi stula but would not cause anorectal agenesis. Premature rupture of the anal membrane would not cause anorectal agenesis.

97
Q

A 12-year-old boy was admitted to the hospital with massive rectal bleeding. Upon inspection, the color of the blood ranged from bright to dark red. The child appeared to be free of any pain. Radiographic examination revealed an ileal (Meckel) diverticulum. Which of the following is the underlying embryologic cause of this condition? ⃣ A. Failure of yolk stalk to regress ⃣ B. Duplication of the intestine ⃣ C. Malrotation of the cecum and appendix ⃣ D. Nonrotation of the midgut ⃣ E. Herniation of the intestines

A

A. Meckel’s diverticulum is a remnant of the yolk stalk. It is usually 2 inches long and 2 feet proximal from the ileocecal junction. Meckel’s diverticulum is prone to ulceration (possibly leading to perforation) that can result in gastrointestinal bleeding. Duplication of the intestine does not predispose the patient to GI bleeding. A subhepatic cecum and appendix is due to failure of the cecal bud to form and results in the absence of an ascending colon. Nonrotation of the midgut could lead to volvulus but is not the most likely cause for GI bleeding. Herniation of intestines can exist without any symptoms; however, if the loop of intestine becomes strangulated, it can lead to gangrene.

98
Q

A 23-year-old pregnant woman visits her gynecologist for her routine ultrasound checkup. Ultrasonographic examination reveals unilateral renal agenesis and oligohydramnios. Which of the following conditions most likely occurred? ⃣ A. Polycystic kidney disease ⃣ B. Degeneration of the mesonephros ⃣ C. Ureteric duplication ⃣ D. Failure of a ureteric bud to form ⃣ E. Wilms tumor

A

D. Failure of the ureteric bud to form results in renal agenesis and oligohydramnios, that is, defi cient production of amniotic fl uid. Polycystic kidney disease is an autosomal recessive disease characterized by spongy kidneys with a multitude of cysts. Degeneration of the mesonephros occurs during normal development; however, a small portion of the meso- nephric tubules may go on to form parts of the urogenital system. Ureteric duplication occurs due to premature division of the ureteric bud and can result in either a double kidney or a duplication of the ureter. Wilms tumor is a malignancy of the kidney that is more common in children than it is in adults; it does not cause renal agenesis.

99
Q

A 15-year-old female was admitted to the hospital with bilateral inguinal masses. Physical examination revealed that she had not begun to menstruate but showed normal breast development for her age. Her external genitalia were feminine, the vagina was shallow, but no uterus could be palpated. Laboratory examination revealed that her sex chromatin pattern was negative. Which of the following is the most likely diagnosis? ⃣ A. Male pseudohermaphroditism ⃣ B. Female pseudohermaphroditism ⃣ C. Androgen insensitivity syndrome ⃣ D. Inguinal hernias ⃣ E. Turner syndrome

A

C. Androgen insensitivity syndrome involves the development of testes and female external genitalia, with a blind-ending vagina and absence of the uterus and uterine tubes. This is consistent with the presenting symptoms. Male pseudohermaphroditism and female pseudohermaphroditism have different presentations from those described and result from 46XY and 46XX genotypes, respectively. Inguinal hernias have nothing to do with absence of the uterus and a negative sex chromatin pattern. Turner syndrome results from a 45X genotype and presents with short stature, webbed neck, congenital hypoplasia of the lymphatics, and shield chest—among other symptoms—and is not consistent with the symptoms described.

100
Q

An 18-year-old female gymnast is admitted to the hospital with pelvic pain. Physical examination reveals that the patient has a history of primary amenorrhea and an imperforate hymen. Which of the following is the most likely explanation for this condition? ⃣ A. Failure of the vaginal plate to canalize ⃣ B. Cervical atresia ⃣ C. Patent processus vaginalis ⃣ D. Androgen insensitivity syndrome ⃣ E. Failure of the sinovaginal bulbs to develop

A

A. The vaginal plate, which arises from the sinovaginal bulbs, undergoes canalization during embryonic development. Failure of canalization results in a persistent vaginal plate and thus imperforate hymen. The hymen is a fold of mucous membrane that covers the opening of the vaginal canal. It is often torn early in life. The processus vaginalis is a tubelike projection of the peritoneum into the inguinal canal that precedes the descent of the testis or round ligament. Both cervical atresia and androgen insensitivity syndrome would result in amenorrhea; however, neither disorder would present with an imperforate hymen because the vaginal canal would still undergo canalization. The sinovaginal bulbs are responsible for the development of the vaginal plate. Failure of development would result in complete absence of the vagina.

101
Q

During a routine gynecologic examination, a 22-year-old female complains of dyspareunia (pain during sexual intercourse). During a pelvic examination a mass of tissue is detected on the lateral wall of the vagina. An ultrasound examination reveals that the abnormal structure is a Gartner duct cyst. From which of the following embryonic structures does this cyst take origin? ⃣ A. Mesonephric tubules ⃣ B. Paramesonephric duct ⃣ C. Urogenital folds ⃣ D. Mesonephric duct ⃣ E. Sinovaginal bulbs

A

D. Gartner ducts cysts, which often appear in the lateral wall of the vagina, are the result of remnants of the mesonephric (wolffi an) duct. The mesonephric duct gives rise to a variety of structures, including the ureter and collecting tubules. In males, the duct eventually forms the ductus deferens and ejaculatory ducts, whereas it often disappears in females. The only remaining traces of the mesonephric ducts are the epoophoron, paroophoron, and Gartner cyst; thus, persistence of this duct can result in formation of these cysts. The mesonephric tubules are elongations of the mesonephric vesicles. These tubules are subsequently invaginated by the glomeruli to form a component of the renal corpuscle. The paramesonephric ducts are responsible for formation of the uterus, cervix, and uppermost aspect of the vagina. The urogenital folds form the labia minora in females and the spongy urethra in males. Sinovaginal bulbs are responsible for the development of the vaginal plate in embryonic development.

102
Q

A 2-day-old male infant is hospitalized in the intensive care unit with acute respiratory distress. Radiographic examination reveals that the patient has anuria, oligohydramnios, and hypoplastic lungs. Facial characteristics are consistent with Potter syndrome. Which of the following is the most likely explanation for these initial fi ndings? ⃣ A. Multicystic dysplastic kidney ⃣ B. Polycystic kidney ⃣ C. Renal agenesis ⃣ D. Wilms tumor ⃣ E. Extrophy of the bladder

A

C. Potter sequence, or Potter syndrome, is a rare autosomal recessive trait and is associated with renal agenesis or hypoplasia. Altered facial characteristics include fl attened nasal bridge, mandibular micrognathia, malformed low-set ears, etc. Absence or lack of proper development of the kidneys causes oligohydramnios, or possibly anhydramnios. Multicystic dysplastic kidney and polycystic kidney are usually secondary to Potter sequence and are therefore not the cause of Potter sequence or oligohydramnios. Wilms tumor is a relatively common renal tumor that presents in children; it is not associated with oligohydramnios and Potter sequence. Extrophy of the bladder is a congenital defect that exposes the posterior surface of the bladder on the exterior of the abdominal wall; there is no indication of this defect in the patient.

103
Q

B. The ureteric bud is responsible for the development of the ureter, and thus an early splitting of the ureteric bud would result in formation of a second ureter on the ipsilateral side. Failure of the ureteric bud to form would cause a complete absence of the ureter, whereas failure of the ureteric bud to branch occurs normally during embryonic development and results in one ureter joined to each kidney. The urorectal septum is a section of tissue of mesenchymal origin that develops between the allantois and hindgut. Failure of this structure to develop would not result in an additional ureter. Finally, a persistent urachus acts as an abnormal fi stula that runs from the bladder to the umbilicus, resulting in urine leaking from the external abnormal wall.

A

A 58-year-old male bricklayer is admitted to the hospital with severe pain that radiates from his lower back to the pubic region. Ultrasound examination reveals that a kidney stone is partially obstructing his right ureter; the examination also indicates the presence of a second ureter on the right side. Which of the following is the most likely cause of this latter fi nding? ⃣ A. Failure of ureteric bud to form ⃣ B. Early splitting of the ureteric bud ⃣ C. Failure of urorectal septum to develop ⃣ D. Persistent urachus ⃣ E. Failure of ureteric bud to branch

104
Q

A 50-year-old female with a long history of heartburn (self-treated with various over-the-counter medications) develops severe epigastric pain and is urgently admitted to the hospital. A gastroscopic examination reveals a small, perforated ulceration in the posterior wall of the stomach body. At surgery, with the patient in supine position, 150 ml of blood-tinged, frothy gray liquid is aspirated from the peritoneal cavity. Where in the peritoneal cavity would liquid most likely fi rst collect when the patient is supine? ⃣ A. Right subphrenic space ⃣ B. Hepatorenal pouch (of Morison) ⃣ C. Left paracolic gutter ⃣ D. Vesicouterine pouch ⃣ E. Rectouterine pouch (of Douglas)

A

B. The hepatorenal pouch (or recess or space) is situated between the liver and both the parietal peritoneum covering the right kidney and suprarenal gland. This recess is the lowest space in the peritoneal cavity in supine patients. Accumulation of fl uid in the peritoneal cavity will ordinarily collect in this pouch. The right subphrenic space is located between the liver and diaphragm. Although this recess is positioned in the appropriate abdominal quadrant, it is not the deepest space within the peritoneal cavity. Fluid may reach the right subphrenic space, but it will not accumulate in this region. The paracolic gutters are grooves lateral to the ascending and descending colons. Though these recesses are potential spaces for fl uid accumulation, they are located inferior to the hepatorenal recess and would not collect fl uid while the patient is supine. The vesicouterine and rectouterine pouch would not be potential spaces for fl uid accumulation in a supine patient, although they provide predictable sites when the patient is upright or ambulating.

105
Q

A 43-year-old female accountant complains of severe epigastric pain and is admitted to the hospital. A gastroscopic examination reveals a small, perforated ulceration in the posterior wall of the greater curvature of her stomach. An upright chest radiograph reveals a small amount of free air in the peritoneal cavity. Where is the air most likely located? ⃣ A. Right subphrenic space ⃣ B. Supravesical space ⃣ C. Paracolic gutters ⃣ D. Vesicouterine pouch ⃣ E. Rectouterine pouch (of Douglas)

A

A. The right subphrenic space is located between the inferior aspect of the diaphragm and the superior surface of the liver. An ulceration in the posterior stomach wall would allow the passage of air from the stomach into the right subphrenic space through an open communication in the omental bursa. The remaining answer choices are located inferior to the site of ulceration and would not accumulate air. The paracolic gutters traverse lateral to the ascending and descending colons. The vesicouterine and rectouterine pouches are located in the pelvic cavity; air would not accumulate in these spaces.

106
Q

A 25-year-old female is admitted to the hospital with sharp pain in the left lower quadrant. Patient history reveals that her last menstrual period was 10 days ago. Transvaginal ultrasound reveals a ruptured cyst on the left ovary. The sonogram also reveals approximately 100 ml of fl uid in the pelvis, which is presumed to represent cyst contents and bleeding from the rupture site. Where is this fl uid most likely located? ⃣ A. Right subphrenic space ⃣ B. Hepatorenal pouch (of Morison) ⃣ C. Paracolic gutters ⃣ D. Vesicouterine pouch ⃣ E. Rectouterine pouch (of Douglas)

A

E. The most likely location of fl uid to accumulate is the pouch of Douglas because it is the lowest point in the pelvis when a patient is standing erect. Additionally, fl uid would accumulate here because the mesoovarian ligament causes the ovary to be located on the posterior aspect of the board ligament. This allows for communication between the fl uid of the ovary and the rectouterine pouch. The right subphrenic space is located inferior to the right side of the diaphragm and superior to the liver. The paracolic gutters are located lateral to the ascending and descending colons. Neither of these locations is a likely site of fl uid accumulation when the patient is standing. The hepatorenal recess is situated between the liver and both the right kidney and suprarenal gland. This recess, also known as the Morison pouch, is the deepest space in the peritoneal cavity in a supine patient but not when the patient sits or stands. The vesicouterine pouch is located between the bladder and uterus and is separated from the ovaries by a double layer of mesentery also known as the broad ligament of the uterus.

107
Q

A 60-year-old man is admitted to the emergency department with severe abdominal pain. Physical examination reveals guarding and rigidity in the abdominal wall. An abdominal CT scan shows a thrombus in an intestinal artery supplying the ileum. Which of the following layers of peritoneum will have to be entered by the surgeon to access the affected vessel? ⃣ A. Parietal peritoneum and the greater omentum ⃣ B. Greater and lesser omentum C. ⃣ Lesser omentum and the gastrosplenic ligament ⃣ D. Parietal peritoneum and the mesentery ⃣ E. Greater omentum and the transverse mesocolon

A

D. Parietal peritoneum and “the mesentery.” The parietal peritoneum lines the abdominal wall, whereas the visceral peritoneum is in intimate contact with organs. The greater omentum extends from the greater curvature of the stomach and covers the midgut. Access to the ileum would require penetration of the parietal peritoneum to enter the peritoneal cavity and interruption of the visceral peritoneum of “the mesentery” covering the thrombosed vessel. Although it would probably be necessary to refl ect the greater omentum for adequate exposure, it would not ordinarily require incision.

108
Q

A 52-year-old male presents to the emergency department complaining of persistent severe right upper quadrant pain for the past 2 hours. During that period of time he felt nauseated, was sweating profusely, and also experienced pain in the posterior aspect of his right shoulder. The pain began shortly after a lunch consisting of “fast food.” Ultrasound examination reveals multiple stones in an infl amed gallbladder with a normal bile duct. Which of the following spinal nerve segments are involved in the shoulder pain, associated with cholecystitis? ⃣ A. C3 to C5 ⃣ B. C5 to C8 ⃣ C. T1 to T4 ⃣ D. T5 to T9 ⃣ E. T10, T11

A

D. Referred pain from cholecystitis is generally referred to the region of the inferior angle of the right scapula. These fi bers are generally from T5 to T9. These sensory fi bers for pain are stimulated by the gallbladder infl ammation because of the proximity of the adjacent structures. C3 to C5 sensory fi bers innervate the shoulder area. The distribution of C5 to C8 is primarily to the upper limb, to the level of the hand; T1 to T4 distribution is to the upper thoracic wall and medial upper arm; T10 and T11 distribution is to the thoracic and abdominal wall, T1 to T4 visceral fi bers for pain are generally associated with referred pain from the heart.

109
Q

During a cholecystectomy on a 64-year-old female, the right hepatic artery was accidentally injured. In addition to bleeding profusely, the lacerated artery was hidden from view by overlying connective tissue and fat. Which of the following procedures would most likely be performed by the surgeon to slow, or perhaps arrest, the blood loss? ⃣ A. Pringle maneuver ⃣ B. Kocher maneuver ⃣ C. Valsalva maneuver ⃣ D. Heimlich maneuver ⃣ E. Placement of a vascular clamp on the porta hepatis

A

A. The Pringle maneuver is a surgical technique employed when the hepatic artery has been accidentally ligated. The hepatoduodenal ligament is clamped off to prevent the passage of blood fl ow through both the hepatic artery and the portal vein. The Kocher maneuver, in which the ascending colon and descending duodenum are refl ected to the left, is often employed to expose and arrest a hemorrhage from the inferior vena cava. The Valsalva maneuver involves stopping the passage of air with the vocal folds to build up intrathoracic pressure, as for coughing or vocalization. The Heimlich maneuver involves sharp, rapid compressions of the xiphisternal region to expel foreign bodies from the trachea. Putting a clamp on the porta would not stop the bleeding but could certainly produce major injury to the numerous vascular elements there.

110
Q

A 53-year-old quality control engineer is admitted to the hospital with severe abdominal pain. The patient’s history and physical examination indicate chronic colonic diverticulitis, with tachycardia and hypotension at the present time. An ultrasonographic examination reveals massive bleeding from the descending colon. Which of the following arteries is most likely to be the source of the hemorrhage? ⃣ A. A branch of the inferior mesenteric ⃣ B. Middle colic ⃣ C. Superior rectal ⃣ D. Inferior rectal ⃣ E. Left gastroepiploic

A

A. The most likely source of this hemorrhage is from the left colic artery. Colonic diverticular disease is the development of blind end sacs from the wall of the colon. Of the selected choices the left colic artery is the only artery supplying a portion of the descending colon. The internal pudendal artery supplies the external genitalia and the perineum. The superior and inferior rectal (hemorrhoidal) arteries supply the rectum and anal canal. The left gastroepiploic artery arises from the splenic artery and supplies the greater curvature of the stomach.

111
Q

The gallbladder of a 51-year-old female patient is characterized by the presence of multiple gallstones, consistent with the diagnosis of cholecystitis. Which of the following tests would be anticipated to be positive in this patient? ⃣ A. Rebound tenderness ⃣ B. Iliopsoas test ⃣ C. Obturator sign ⃣ D. Murphy’s sign ⃣ E. Cough tenderness

A

D. Murphy’s sign is a specifi c test designed to detect and diagnose problems in the upper right abdominal quadrant. It is classically used to diagnose problems of the gallbladder by pressing deeply under the right costal margin and asking the patient to breathe deeply. This causes sharp pain in the patient with cholecystitis. Rebound tenderness is pain experienced when applied pressure is removed quickly from a location on the body. It is not specifi c to areas of the body and is not indicative of problems in the upper right quadrant. The iliopsoas test is generally used to diagnose appendicitis because fl exing the iliopsoas muscle group applies pressure to the ascending colon and appendix. When the appendix is infl amed, this pressure elicits pain. The obturator sign is used to diagnose irritation of the obturator internus muscle and would not elicit a positive test with the patient’s current symptoms. Finally, cough tenderness would not be present because it is generally associated with hernias and problems associated with rises in intraabdominal pressure.

112
Q

A 35-year-old man is admitted to the hospital with a small-caliber bullet wound to the left upper quadrant of the abdomen. Radiographic examination reveals profuse intraperitoneal bleeding. An emergency laparotomy is performed, and the source of bleeding appears to be a vessel within the lesser sac. Which of the following ligaments would most likely be transected to gain adequate entry to the lesser sac? ⃣ A. Coronary ⃣ B. Gastrosplenic ⃣ C. Splenorenal ⃣ D. Gastrocolic ⃣ E. Hepatoduodenal

A

D. The lowest point of the lesser sac occurs at the intersection of the gastrocolic ligament and transverse colon. Bleeding would travel to the lowest point of the lesser sac. Access to this area would require entry through the gastrocolic ligament, which extends from the greater curvature of the stomach to the transverse colon. The hepatogastric and hepatoduodenal ligaments attach between the liver and the lesser curvature of the stomach and the fi rst part of the duodenum, respectively. Dividing the hepatogastric ligament provides entry into the lesser omentum, but this is not the most effi cient access point because it does not provide the best exposure of the inferior aspect of the lesser sac.

113
Q

A 45-year-old woman is admitted to the hospital with rectal bleeding. Physical examination, including a rectal examination, reveals an abnormal mass of tissue protruding below the pectinate line. Biopsy reveals the presence of an adenocarcinoma. Which of the following groups of lymph nodes would fi rst receive lymph from the area of pathology? ⃣ A. Internal iliac ⃣ B. External iliac ⃣ C. Middle rectal ⃣ D. Superfi cial inguinal ⃣ E. Deep inguinal

A

D. Superfi cial inguinal nodes. The external anal sphincter is skeletal muscle of the anal canal. This suggests that the carcinoma is originating from the anal canal. The results of the biopsy support the fi nding that the carcinoma most likely occurred below the pectinate line where you would fi nd squamous cells of the anal canal. The anal canal primarily drains to the superfi cial inguinal lymph nodes. The inferior rectum above the pectinate line drains into the internal iliac. The superior aspect of the rectum drains into the middle rectal lymph nodes. The external iliac nodes primarily drain lower limb, pelvic, and deep peritoneal structures. The deep inguinal lymph nodes drain the glands of the clitoris and penis and the superfi cial inguinal nodes.

114
Q

A 53-year-old man is admitted to the hospital with rectal bleeding. Physical examination, including a rectal examination, reveals an abnormal mass of tissue protruding from an area superior to the external anal sphincter, superior to the pectinate line. Biopsy reveals the presence of an adenocarcinoma. Which of the following groups of lymph nodes would fi rst receive lymph from the cancerous area? ⃣ A. Internal iliac ⃣ B. External iliac ⃣ C. Middle rectal ⃣ D. Superfi cial inguinal ⃣ E. Deep inguinal

A

A. The lymphatics of the inferior rectum above the pectinate line drain into the internal iliac nodes. Below the pectinate line, lymphatics of the anal canal will primarily drain into the superfi cial inguinal (horizontal) nodes. The external iliac nodes primarily drain lower limb, pelvic, and deep peritoneal structures. The deep inguinal lymph nodes primarily drain the glands of the clitoris and penis, receiving lymph also from the superfi cial inguinal nodes. The superior aspect of the rectum is drained by the middle rectal nodes, lymph from which eventually fl ows into the lumbar nodes.

115
Q

A 32-year-old male is admitted to the emergency department with severe esophageal refl ux. Radiographic examination reveals that the patient has a hiatal hernia, and a surgical procedure is scheduled. Which of the following landmarks would be the most useful to distinguish between sliding and paraesophageal hiatal hernias? ⃣ A. Sliding hernias possess a normal gastroesophageal junction. ⃣ B. In sliding hernias the gastroesophageal junction is displaced. ⃣ C. Paraesophageal hernias have a displaced gastroesophageal junction. ⃣ D. In paraesophageal hernias the antrum moves into the stomach corpus. ⃣ E. In paraesophageal hernias the antrum and the cardia move into the body of the stomach.

A

B. In sliding hernias the gastroesophageal junction is displaced. Diaphragmatic hernias of the esophagus can be characterized by analyzing the gastroesophageal junction. In sliding hernias the gastroesophageal junction is displaced anteriorly into the mediastinum. The paraesophageal hernia is generally characterized by herniation of the stomach into the mediastinum; however, the gastroesophageal junction remains fi xed. In paraesophageal hernias the fundus herniates into the stomach, but the antrum does not.

116
Q

A 43-year-old man is admitted to the emergency department with complaints of intense abdominal pain. Radiographic examination reveals a right subphrenic abscess that extends to the midline. Which of the following structures would most likely be in a position to retard the spread of the abscess across the midline? ⃣ A. Round ligament ⃣ B. Falciform ligament ⃣ C. Coronary ligament ⃣ D. Hepatoduodenal ligament ⃣ E. Gastroduodenal ligament

A

B. The falciform ligament separates the subphrenic spaces into right and left recesses and extends between the liver and the anterior abdominal wall. Because of its location and attachments, it would serve to stop the spread of such an abscess from one side to the other. The location of the other answer choices makes them insuffi cient in limiting the spread of an abscess. The round ligament of the liver is the obliterated remains of the umbilical vein and lies in the free margin of the falciform ligament ascending from the umbilicus to the inferior surface of the liver. The coronary ligament encloses the bare area of the liver and forms the triangular ligaments. The hepatoduodenal and hepatogastric ligaments attach the liver to the duodenum and stomach, respectively. Together they form the lesser omentum.

117
Q

7 A 21-year-old football player is admitted to the emergency department with intense back pain. Physical examination shows that his left lower back is bruised and swollen. He complains of sharp pain during respiration. A radiograph reveals a fracture of the eleventh rib on the left side. Which of the following organs would be the most likely to sustain injury at this site? ⃣ A. Spleen ⃣ B. Lung ⃣ C. Kidney ⃣ D. Liver ⃣ E. Pancreas

A

C. The kidney lies at the twelfth rib, and problems with pain associated with respiratory processes would result from the ribs being injured. The spleen lies under the left side of the liver superior to the kidneys, adjacent to ribs 9 to 11. The lungs are located completely within the thoracic cavity above the level of the twelfth rib. The liver is located on the right side of the body and lies around the level of the fi fth to tenth ribs. The pancreas is predominantly located in the middle of the body more medial to the kidneys at the level of the eleventh to twelfth ribs.

118
Q

A 46-year-old man is admitted to the hospital with a rather large but painless mass on his right groin. During physical examination it is noted that the inguinal lymph nodes are hard and palpable. A lymph node biopsy reveals the presence of malignant cells. Which of the following locations would be the most likely primary source of carcinoma? ⃣ A. Prostate ⃣ B. Bladder ⃣ C. Testis ⃣ D. Anal canal ⃣ E. Sigmoid colon

A

D. A malignancy of the anal canal would drain into the inguinal lymph nodes, specifi cally the superfi cial lymph nodes. The internal iliac lymph nodes receive drainage from the rectum, the uterus, the prostate gland, and the bladder. The testes, though located in the scrotum, external to the abdominopelvic cavities, drain into the lumbar nodes located on the anterior aspect of the aorta. The sigmoid colon drains into the inferior mesenteric lymph nodes.

119
Q

A 54-year-old man is admitted to the hospital with vomiting and severe weight loss. Physical examination reveals that the umbilical and epigastric regions are tender and painful. A CT scan examination reveals a massive tumor originating from the third part of the duodenum. Which of the following structures is more likely to be compressed or invaded by the tumor? ⃣ A. Common bile duct ⃣ B. Portal vein ⃣ C. Superior mesenteric artery ⃣ D. Gastroduodenal artery ⃣ E. Posterior superior pancreaticoduodenal artery

A

C. The superior mesenteric artery arises from the aorta, behind the neck of the pancreas, at the level of L1 within the abdominal cavity, and traverses inferiorly across the anterior surface of the third part of the duodenum. As it crosses the uncinate process of the pancreas and duodenum, this artery could readily be affected by a tumor in the immediate area. The common bile duct and the portal vein course anteriorly from the head of the pancreas and therefore are associated with the fi rst part of the duodenum. The gastroduodenal artery supplies the duodenum, the head of the pancreas, and the greater curvature of the stomach and is a branch of the common hepatic artery. This artery does not cross the third part of the duodenum. The posterior superior pancreaticoduodenal courses posteriorly over the head of the pancreas and supplies both the head of the pancreas and the second portion of the duodenum. It is therefore more associated with the second part of the duodenum.

120
Q

A 24-year-old woman is admitted to the hospital with lower abdominal pain. A CT examination reveals an abnormal mass occupying the left adnexa in the pelvis. During the surgical procedure the ureter and the structures immediately medial to the ureter are identifi ed. Which of the following vascular structures crosses the ureter just lateral to the cervix of the uterus? ⃣ A. Middle rectal artery ⃣ B. Superior vesical artery ⃣ C. Internal pudendal vein ⃣ D. Uterine artery ⃣ E. Gonadal vein

A

D. Vascular structures situated immediately medial to the ureter are often subject to ligation during surgical procedures. The ureter is crossed by the uterine artery an inch or so lateral to the cervix and must be identifi ed and avoided, in ligating the uterine vessels. The middle rectal artery arises from the internal iliac artery and passes dorsal to the uterus and the rectum. The superior vesical artery arises from the umbilical artery anteriorly in the pelvis. The internal pudendal vein enters the pelvis through the greater sciatic foramen and terminates in the internal iliac vein, laterally at the pelvic wall. The gonadal vein in the female passes to the ovary by way of the infundibulopelvic ligament, near the pelvic brim.

121
Q

A 32-year-old woman is admitted to the hospital with cramping abdominal pain around her umbilicus and vomiting for the previous 2 days. Radiographic studies indicate numerous stones in the gallbladder and air accumulation in the gallbladder and biliary tree. At which of the following places will an obstructive stone most likely be found? ⃣ A. Jejunum ⃣ B. Terminal ileum ⃣ C. Common bile duct ⃣ D. Duodenum ⃣ E. Hepatic duct

A

B. Pain in the umbilical region can be indicative of referred pain from the large intestine. Gallstones can ulcerate through the wall of the fundus of the gallbladder and into the transverse colon, or through the wall of the body of the gallbladder into the duodenum. The stone would then most likely be entrapped at the ileocecal junction, possibly leading to an intestinal obstruction. This could lead predictably to the pain, cramping, and vomiting experienced by the patient. The radiographic results suggest that the biliary trees are clear (indicated by the presence of air) and rule out the common bile duct or hepatic duct as potential sites of gallstone obstruction. Because the gallstones would pass thereafter through the duodenum, this would not be the site of blockage. Because the gallstones could pass freely through the intestine as far distally as the ileocecal junction, it would be unlikely that they would accumulate in the jejunum.

122
Q

A 37-year-old woman is admitted to the hospital with signs of cholecystitis. A physical examination confi rms the initial diagnosis and a cholecystectomy is planned. Which of the following landmarks will best describe the precise location of the gallbladder with respect to the body wall? ⃣ A. The intersection of the right linea semilunaris with the ninth costal cartilage ⃣ B. The intersection of the right linea semilunaris with the intertubercular plane ⃣ C. To the right of the epigastric region ⃣ D. Superiorly to the umbilical region ⃣ E. Upper right quadrant

A

A. The intersection of the right linea semilunaris with the ninth costal cartilage in the right upper quadrant is associated typically with the point of contact of the gallbladder fundus with the anterior abdominal wall. The anatomic quadrants provide a useful tool for understanding the location of various anatomic structures and viscera located within the body. The linea semilunaris runs parallel with the lateral border of the rectus sheath and is a prominent landmark for surface anatomy. The upper right quadrant is the correct anatomic region for the location of the gallbladder as mentioned earlier; however, this choice is too general and would not be the best answer choice. Similarly, the intersection of the right semilunaris with the right intertubercular plane is situated in the upper right quadrant; however, this is not the most precise location of the gallbladder. The epigastric region is located superior to the umbilical region. The contents of the epigastric region are the left portion of the liver and a portion of the stomach. The right hypochondriac region, the anatomic region situated to the right of the epigastric region, is located superior and lateral to the gallbladder and would thus not be the appropriate answer choice.

123
Q

A 45-year-old male is admitted to the hospital with jaundice. This patient has a long history of alcoholism. Radiographic studies reveal ascites, portal hypertension, and liver cirrhosis. Which of the following veins are likely to be responsible for the production of ascitic fl uid? ⃣ A. Direct portal vein tributaries ⃣ B. Anastomosing vessels between parietal veins and veins of retroperitoneal intestine ⃣ C. Paraumbilical veins ⃣ D. Esophageal submucosal anastomoses with azygos tributaries ⃣ E. Superior rectal left gastric and middle rectal veins

A

B. Veins of the body wall and veins of the retroperitoneal intestinal organs are interconnected by extensive, thin-walled, anastomosing vessels. These veins tend to leak watery fl uid in portal hypertension. Ascites is the accumulation of such fl uid in the peritoneal cavity from these collateral veins. Ascites can occur within the veins of the body wall or veins of the retroperitoneal organs. An anastomosis between the epigastric veins and the paraumbilical veins would lead to possible contribution to caput medusae but would not likely result in signifi cant ascites. Esophageal varices are due to expansion of submucosal esophageal veins from portal hypertension, resulting from anastomoses between the left gastric vein and the esophageal veins. These anastomoses would produce varices, perhaps with profuse bleeding, rather than ascites. Further, ascites occur within the peritoneal cavity. Superior rectal, left gastric, and the middle rectal veins all contribute to the portal-systemic anastomoses, but these anastomoses form varices or hemorrhoids, not ascites. The portal vein would not lead to ascites because it is not involved directly with any caval anastomoses. GAS 339, 392; GA 172

124
Q

A 45-year-old man is admitted to the hospital with a palpable and painful mass at his groin that is exacerbated when he stands erect or physically exerts himself. Physical examination indicates the probability of a direct inguinal hernia, a diagnosis that is confi rmed laparoscopically. Which of the following is the most likely cause of this type of inguinal hernia? ⃣ A. Defective transversalis fascia around the deep inguinal ring ⃣ B. Defective peritoneum around the deep inguinal ring ⃣ C. Defective aponeurosis of external abdominal oblique muscle ⃣ D. Defective extraperitoneal connective tissue ⃣ E. Defective aponeurosis of transversus abdominis muscle

A

E. The transversus abdominis aponeurosis and transversalis fascia form a signifi cant portion of the posterior wall of the inguinal canal and the lower part of the inguinal triangle (of Hesselbach). Gradual weakness or attrition of tissues in the posterior wall provides the likelihood of egress of a direct inguinal hernia. A patent processus vaginalis at the deep inguinal ring, or expansion of the deep inguinal ring, with stretching of the transversalis fascia there, can contribute to the formation of indirect inguinal hernias. Weakness of the transversalis fascia by itself is not a key feature of inguinal herniation, nor is weakness of the peritoneum, or defects in the aponeuroses of the external or internal oblique muscles.

125
Q

A 22-year-old female is admitted to the hospital with a complaint of intense periumbilical pain. Physical examination indicates a strong possibility of appendicitis. Shortly before an appendectomy is to be performed, the infl amed appendix ruptures. In which area would the extravasating blood and infectious fl uids from the appendiceal region most tend to collect if the patient was sitting upright? ⃣ A. Subphrenic space ⃣ B. Hepatorenal recess (pouch of Morison) ⃣ C. Rectouterine recess (pouch of Douglas) ⃣ D. Vesicouterine space ⃣ E. Subhepatic space

A

C. The rectouterine pouch (of Douglas) is the lowest recess of the female abdominopelvic cavity when the woman is standing or sitting upright. Any fl uid accumulation in this cavity will settle in the rectouterine pouch due to it being the most dependent or inferior space. The subphrenic space would likely not collect fl uid because of its location in the superior abdominal cavity, which does not tend to collect fl uid from the pelvis. The hepatorenal pouch (of Morison) is located in the right posterosuperior aspect of the abdominal cavity, far from the pelvic cavity. The vesicouterine space is a recess that is similarly located in the lower portion of the abdomen between the urinary bladder and uterus, but it is slightly superior to the pouch of Douglas and separated from it and the pathway of the leaking fl uid by the broad ligament of the uterus. The broad ligament tends to prevent the collection of fl uids in the vesicouterine pouch.

126
Q

The 22-year-old female patient cried out in agony from the pain of her burst appendix. Which of the following structures contain the neuronal cell bodies of the pain fi bers from the appendix? ⃣ A. Sympathetic chain ganglia ⃣ B. Celiac ganglion ⃣ C. Lateral horn of the spinal cord ⃣ D. Dorsal root ganglia of spinal nerves T8 to T10 ⃣ E. Dorsal root ganglia of spinal nerves L2 to L4

A

D. The dorsal root ganglia contain all cell bodies of sensory neurons from the body wall and limbs. Afferent fi bers from the appendix travel in T8 to T10. The sympathetic chain contains postganglionic sympathetic cell bodies that are targeted to smooth muscle and glands of the viscera and heart muscle. The greater splanchnic nerve (T5 to T9) carries preganglionic sympathetic axons to the celiac ganglion, which is formed by postganglionic sympathetic neurons. The lateral horn of the spinal cord is found in levels T1 to L2 and contains preganglionic sympathetic cell bodies.

127
Q

A 30-year-old woman complains of weakness and fatigability over the past 6 months. She has a 3-month acute history of severe hypertension that has required treatment with antihypertensive medications. Radiographic examination reveals a tumor of her right suprarenal gland. The patient is diagnosed with a pheochromocytoma (tumor of the adrenal medulla) and is scheduled for a laparoscopic adrenalectomy. Which of the following nerve fi bers will need to be cut when the adrenal gland and tumor are removed? ⃣ A. Preganglionic sympathetic fi bers ⃣ B. Postganglionic sympathetic fi bers ⃣ C. Somatic motor fi bers ⃣ D. Postganglionic parasympathetic fi bers ⃣ E. Preganglionic parasympathetic fi bers

A

A. The preganglionic sympathetic fi bers running to the adrenal gland would be cut during adrenalectomy for they synapse on catecholamine-secreting cells within the adrenal medulla. Unlike the normal route of sympathetic innervation, which is to fi rst synapse in a sympathetic ganglion and then send postganglionic fi bers to the target tissue, the chromaffi n cells of the adrenal gland are innervated directly by preganglionic sympathetic fi bers. This is because the chromaffi n cells are embryologically postganglionic neurons that migrate to the medulla and undergo differentiation. The adrenal gland receives no other recognized types of innervation; therefore, the remaining answer choices are all incorrect.

128
Q

A 55-year-old man is admitted to the hospital for his annual checkup. An ultrasound examination reveals a tumor that has incorporated the right common iliac artery and compressed the vein that lies posterior to it. Doppler ultrasound studies give evidence of the development of a deep venous thrombosis that could block venous return from the left lower limb, causing ischemia and pain. Which of the following vessels is most likely to be involved in the production of the deep venous thrombosis? ⃣ A. Inferior vena cava ⃣ B. Right renal vein ⃣ C. Left testicular vein ⃣ D. Left common iliac vein ⃣ E. Right common iliac vein

A

D. The left common iliac vein lies posterior to the right common iliac artery. Compression of the vein in this location is a frequent cause of deep venous thrombosis of the left lower limb; that is, the venous drainage of the lower limb is obstructed. This can cause extreme pain, together with ischemia of the limb that, in some untreated cases, can lead to amputation of the limb or death. The inferior vena cava would not be compressed, for it has branched superior to the location of the tumor. The renal veins would not be compromised because these veins extend from the kidneys to the inferior vena cava quite far above the blockage. The testicular veins pass lateral to the area of obstruction, with the right gonadal vein passing superiorly to join the inferior vena cava, and the left gonadal vein terminating in the left renal vein. The right common iliac vein passes freely about the pelvic brim after taking origin from the inferior vena cava and would not be subjected to compression by the right common iliac artery. The subcostal nerve exits the spinal cord at the level of T12 and runs into the abdominal cavity behind the diaphragm on top of the quadratus lumborum.

129
Q

A 48-year-old woman is admitted to the emergency department with a complaint of severe abdominal pain. Radiographic examination reveals advanced carcinoma of the head of the pancreas. A celiac plexus block is performed to relieve her pain. Which of the following best describes the nerve structures that are most likely to be present in the celiac ganglion? ⃣ A. Preganglionic parasympathetic and somatic motor fi bers ⃣ B. Postganglionic parasympathetic and visceral afferent fi bers ⃣ C. Postganglionic sympathetic and visceral afferent fi bers ⃣ D. Pre- and postganglionic sympathetic, preganglionic parasympathetic, and visceral afferent fi bers ⃣ E. Preganglionic sympathetic, preganglionic parasympathetic, and visceral afferent fi bers

A

D. Preganglionic and postganglionic sympathetics, preganglionic parasympathetic, and visceral afferent fi bers are present within the celiac ganglion. The cell bodies of postganglionic sympathetic fi bers are contained within the celiac ganglion and their axons pass to upper abdominal organs. Preganglionic parasympathetic nerves also run through the ganglion but do not synapse within the ganglion; therefore, there are no postganglionic parasympathetic nerves in the ganglion. The preganglionic parasympathetic fi bers are extensions from the right vagal trunk and run within the preaortic plexus. No somatic motor fi bers are present within this ganglion. Running through all of the abdominal ganglia are also visceral afferent fi bers passing superiorly to reach the spinal cord at spinal nerve levels T5 to L2. Answer B is incorrect because there are no postganglionic parasympathetic fi bers running within the ganglion or the celiac plexus. Postganglionic parasympathetic nerves arise from terminal ganglia located upon, or within the wall of, target organs. Answers C and E are incorrect because they do not include postganglionic sympathetic cell bodies and their axons, which also run though the celiac ganglion.

130
Q

A 21-year-old woman is admitted to the hospital with a complaint of severe pain radiating from her lower back toward and above the pubic symphysis. Ultrasound examination reveals that a kidney stone is partially obstructing her right ureter. Which of the following nerves is most likely responsible for conducting the sensation of pain? ⃣ A. Subcostal ⃣ B. Iliohypogastric ⃣ C. Ilioinguinal ⃣ D. Lateral femoral cutaneous ⃣ E. Obturator

A

C. The ureter is innervated by sympathetic and parasympathetic fi bers in the ureteric plexus. General visceral afferent fi bers in the ureteric plexus follow sympathetic fi bers from spinal cord levels T11 to L2; therefore, pain from these fi bers will be referred to nerves at these levels. The iliohypogastric nerve receives fi bers from T12 and L1 and innervates the skin over the pubic symphysis. The ilioinguinal nerve receives fi bers from L1 and innervates the skin from the iliac crest to the upper portions of the labia. The subcostal nerve innervates skin at the T12 dermatome level. The lateral cutaneous branch of the iliohypogastric nerve contains fi bers from T12-L1 and supplies the anterior superior gluteal skin. The lateral femoral cutaneous nerve contains fi bers from L2 and L3 spinal cord levels and innervates the skin over the lateral thigh. The obturator nerve contains fi bers from L2 to L4 spinal cord levels and innervates the major adductors of the thigh

131
Q

A 42-year-old woman is admitted to the hospital with an acutely painful abdomen. Radiographic examination reveals penetration of a posterior duodenal ulcer resulting in intraabdominal bleeding. Which of the following arteries is most commonly eroded by this type of ulcer? ⃣ A. Gastroduodenal artery ⃣ B. Superior mesenteric ⃣ C. Posterior superior pancreaticoduodenal ⃣ D. Posterior inferior pancreaticoduodenal ⃣ E. Right gastric

A

C. Perforation of a posterior duodenal ulcer most commonly damages the posterior superior pancreaticoduodenal artery. This artery branches from the inferior aspect of the gastroduodenal artery. The superior mesenteric artery does not lie directly beneath the duodenum and is not likely to be damaged. The posterior inferior pancreaticoduodenal artery branches from the superior mesenteric artery and lies too far inferior to be damaged. The right gastric artery branches from the proper hepatic artery and runs along the pyloric portion of the lesser curvature of the stomach.

132
Q

A 37-year-old female court stenographer is admitted to the hospital with a complaint of intense pain in her abdomen. Radiographic examination reveals penetration of an anterior duodenal ulcer. Which of the following conditions will most probably occur? ⃣ A. Bleeding from gastroduodenal artery ⃣ B. Bleeding from superior mesenteric artery ⃣ C. Bleeding from posterior superior pancreaticoduodenal artery ⃣ D. Bleeding from posterior inferior pancreaticoduodenal artery ⃣ E. Peritonitis

A

E. A perforating ulcer in the anterior wall of the duodenum is likely to cause peritonitis. The duodenum is covered anteriorly by a layer of peritoneum. The gastroduodenal and posterior superior pancreaticoduodenal arteries lie posterior to the duodenum and are not likely to be damaged. The posterior inferior pancreaticoduodenal artery is a branch of the superior mesenteric artery and neither of these arteries is likely to be damaged because they lie inferior to the duodenum. Perforating ulcers pierce the duodenum or stomach anteriorly; penetrating ulcers pierce the duodenum or stomach posteriorly.

133
Q

A 56-year-old male is diagnosed with midgut volvulus and intestinal ischemia. A laparotomy is performed to release the obstruction of the intestines. Which of the following structures is used as a landmark to determine the position of the duodenojejunal junction? ⃣ A. Superior mesenteric artery ⃣ B. Inferior mesenteric artery ⃣ C. Vasa recta ⃣ D. Suspensory ligament of the duodenum (ligament of Treitz) ⃣ E. Phrenocolic ligament

A

D. The suspensory ligament of the duodenum (ligament of Treitz) originates from the right crus of the diaphragm and is attached to the fourth part of the duodenum at the duodenojejunal junction. This ligament is commonly used as a palpable landmark during abdominal surgeries. The superior and inferior mesenteric arteries and the vasa recta are highly variable and cannot be used as reliable landmarks. Ladd bands connect the cecum to the abdominal wall and can obstruct the duodenum in cases of malrotation of the intestine. These bands can be surgically divided to treat malrotation of the intestine, but Ladd bands are not used as a common landmark.

134
Q

A 4-month-old female infant is admitted to the hospital with cyanosis. Physical examination reveals decreased breath sounds, abdominal sounds in the thorax, and respiratory distress. A radiographic examination reveals a posterolateral defect of the diaphragm and abdominal contents in the left pleural cavity. Which of the following is the most likely cause of this defect? ⃣ A. Absence of a pleuropericardial fold ⃣ B. Absence of musculature in one half of the diaphragm ⃣ C. Failure of migration of diaphragm ⃣ D. Failure of the septum transversum to develop ⃣ E. Failure of pleuroperitoneal fold to close

A

E. Failure of fusion of the pleuroperitoneal folds can cause herniation of abdominal contents into the thorax (congenital diaphragmatic hernia), most commonly on the left side. This defect can impair lung function, causing respiratory distress and cyanosis. The absence of the pleuropericardial fold would cause communication between the pericardial sac and the pleural cavity of the lung and would not lead to the symptoms described. The musculature of the diaphragm is derived from the third to fi fth cervical myotomes. Absence of musculature in one half of the diaphragm (eventration of the diaphragm) would cause paradoxical respiration. The downward migration of the diaphragm is due to the elongation of the posterior body wall and is not likely to lead to this patient’s condition. Failure of the septum transversum to develop would cause an absence of the central tendon of the diaphragm and is not normally associated with congenital diaphragmatic hernia.

135
Q

A 58-year-old male complains of sharp epigastric pain, most commonly felt just after a large meal. He is tender to palpation at the xiphisternal junction. Barium swallow exams and dye injections (HIDA scan) to test gallbladder functions are negative. Ultrasound reveals that a portion of the greater omentum is trapped at its entry to the thorax between the xiphoid process and the costal margin on the right. What is the most likely diagnosis of this condition? ⃣ A. Bochdalek hernia ⃣ B. Sliding esophageal hernia ⃣ C. Morgagni hernia ⃣ D. Cholecystitis ⃣ E. Hiatal hernia

A

C. Both Morgagni and Bochdalek hernias are due to defects in the pleuroperitoneal membrane. Morgagni hernia is normally found retrosternally just lateral to the xiphoid process and, if severe, can cause respiratory distress. More commonly, it is perceived as a sharp, epigastric pain that can be confused with several other maladies. In this case, the barium test rules out esophageal problems; the HIDA scan injection to study the gallbladder and biliary apparatus is negative also. Bochdalek hernia can cause similar symptoms but is due to a posterolateral herniation and would not be near the xiphoid process. A congenital hiatal hernia occurs when part of the stomach herniates into the thoracic cavity and can be caused by a shortened esophagus. This type of hernia would not present between the xiphoid process and costal margin.

136
Q

A 62-year-old woman is admitted to the emergency department with abdominal pains of uncertain origin. A CT scan reveals an aortic aneurysm affecting the origin of the superior mesenteric artery, resulting in ischemia to an abdominal organ. Which of the following organs is most likely affected? ⃣ A. Ileum ⃣ B. Transverse colon ⃣ C. Spleen ⃣ D. Stomach ⃣ E. Duodenum

A

A. The ileum can become ischemic when arterial supply from the superior mesenteric artery is compromised. The superior mesenteric artery arises from the aorta posterior to the neck of the pancreas. It descends across the third part of the duodenum and enters the root of the mesentery behind the transverse colon. This artery gives origin to the following branches: inferior pancreaticoduodenal artery, middle colic artery, ileocolic artery, right colic artery, and intestinal arteries. The ileocolic artery descends behind the peritoneum toward the right and ends by dividing into the ascending colic artery, anterior and posterior cecal arteries, the appendicular artery, and ileal branches. The ileum is supplied by the ileal branches, which do not have any anastomoses with another major source vessel. The transverse colon is supplied by the marginal artery, which possesses anastomoses of the right colic artery arising from the superior mesenteric artery, and the left colic artery arising from the inferior mesenteric artery. The spleen, stomach, and duodenum are all supplied by branches of the celiac trunk, which arise from the abdominal aorta just below the aortic hiatus of the diaphragm.

137
Q

A 41-year-old man entered the emergency department with abdominal trauma after his motorcycle was struck by a hit-and-run automobile driver. One week following emergent surgery the patient was released from the hospital. Two months postoperatively the patient complained of abdominal pain. A CT scan examination demonstrated an internal hernia in which the hepatic fl exure of the colon had herniated through the epiploic (omental) foramen (of Winslow). Gastrointestinal veins appeared to be markedly dilated, including the veins forming anastomoses between the portal and caval systems (veins of Retzius). Which of the following structures is most likely compressed? ⃣ A. Portal vein ⃣ B. Inferior vena cava ⃣ C. Hepatic artery ⃣ D. Common bile duct ⃣ E. Cystic duct

A

A. The portal vein is compressed in its passage through the hepatoduodenal ligament, the anterior border of the omental (epiploic) foramen (of Winslow). The veins of Retzius are located along the sides of the abdominal walls and communicate between tributaries of retroperitoneal parts of the gastrointestinal tract and veins of the body wall. In portal hypertension the portal blood cannot pass freely through the liver, and the portal-caval tributaries and their anastomoses become dilated. The inferior epigastric veins anastomose with the paraumbilical veins, which is the fi rst branch off the hepatic portal vein. These would be the fi rst affected in portal hypertension. The inferior vena cava is the main route of blood return to the right atrium and is posterior to the omental (epiploic) foramen (of Winslow); it would not likely be compressed due to herniation through the foramen. Compression of the proper hepatic artery in the hepatoduodenal ligament would not result in dilation of veins of Retzius but could conceivably diminish blood supply to the gallbladder and liver. Common bile duct compression would result in jaundice and increased serum bilirubin. The cystic duct joins with the hepatic duct to form the common bile duct. Compression of this would lead to an infl amed gallbladder (cholecystitis).

138
Q

A 48-year-old woman visited the outpatient clinic with a complaint of lower left quadrant pain that had persisted for the previous 3 months. Laboratory examinations revealed that the patient had blood in her stools. A colonoscopy gave evidence of diverticulosis that had been affecting the distal part of the descending colon. To which of the following dermatomes would pain have most likely been referred? ⃣ A. T5 to T9 ⃣ B. T10 to L1 ⃣ C. L1, L2 ⃣ D. L1 to L4 ⃣ E. T10 to L2

A

C. A dermatome is an area of skin that is supplied by a single spinal nerve. The descending colon receives its visceral sensory supply for pain from spinal segments L1 and L2. Injury to the descending colon can cause referral of pain to the corresponding dermatomes. Spinal segments T6 to T10 supply upper abdominal organs, including the pancreas and the duodenum. L1 to L4 supply the rectum, bladder, and uterus. T6 to L2 supply portions of all abdominal viscera.

139
Q

A 61-year-old man is admitted to the emergency department with abdominal pain and a 2-day history of vomiting. Physical examination reveals a colicky abdominal pain in the right lower quadrant, with abdominal distention. Upon auscultation, episodes of pain were associated with rushes, gurgling, and tinkling sounds. A CT scan examination reveals a mechanical obstruction of the bowel. Which of the following parts of the gastrointestinal tract is most likely obstructed? ⃣ A. Hepatopancreatic ampulla of Vater ⃣ B. Duodenal bulb ⃣ C. Proximal ileum ⃣ D. Pyloric sphincter ⃣ E. Ileocecal junction

A

E. A gallstone ileus occurs when a gallstone (cholelith) ulcerates through the wall of the body of the gallbladder and into the duodenum. In this case, gallstones became lodged in the ileocecal region. Obstruction in the ileocecal junction can produce pain that mimics appendicitis. However, bowel sounds will be exaggerated above the obstruction and absent distal to the obstruction. This obstruction would require surgical correction. The hepatopancreatic ampulla (of Vater) is the location where the pancreatic and bile ducts join before entering the duodenum. An obstruction here would cause jaundice, and radiating pain would be localized into the right upper quadrant, with referred pain to the scapula. The duodenal cap, or bulb, is proximal to the entrance of the common bile duct, and any obstruction would be distal to this point. The ileum is not a likely site for gallstone obstruction. The pyloric sphincter surrounds the pyloric orifi ce and controls the rate of emptying of the stomach contents into the duodenum. This location is proximal to the entrance of gallbladder contents into the small intestine.

140
Q

A 43-year-old woman is admitted to the emergency department with esophageal pain and hematemesis after swallowing a fi sh bone. An endoscopic examination reveals perforation of the intraabdominal portion of the esophageal wall. Which of the following arteries is most likely injured? ⃣ A. Branches of left gastric ⃣ B. Bronchial ⃣ C. Thoracic intercostal ⃣ D. Branches of right gastric ⃣ E. Right inferior phrenic

A

A. The upper and intermediate portions of the esophagus receive blood supply from three branches of the aorta: the inferior thyroid, bronchial, and esophageal arteries; the lower portion of the esophagus is supplied by the inferior phrenic and left gastric arteries. The lowest part of the esophagus, below the diaphragm, is supplied by the left gastric artery. Perforation to this area could easily injure this artery. The bronchial artery supplies a small section of the esophagus inferior to the level of the carina (T4). The thoracic intercostal arteries supply intercostal spaces and do not contribute to esophageal arterial supply. The right gastric arises from the common hepatic artery and supplies the pyloric part of the lesser curvature of the stomach. The inferior phrenic supplies the portion of the esophagus just superior to the diaphragm.

141
Q

A 42-year-old patient is admitted to the hospital after suffering a ruptured spleen in a skiing accident. Physical examination reveals intense pain that radiates to the region of the left shoulder, presumably due to irritation of the diaphragm. Which of the following signs best describes this condition? ⃣ A. Mittelschmerz ⃣ B. Kerh sign ⃣ C. Rovsing sign ⃣ D. Psoas sign ⃣ E. Obturator sign

A

B. Kerh sign is a clinical indication of a ruptured spleen and is characterized by intense radiating pain to the top of the left shoulder. Mittelschmerz can occur in the middle of a woman’s menstrual cycle when the graafi an follicle ruptures and the ovum is released from the ovary. Rovsing sign is a clinical indicator of gallbladder infl ammation and pain referred to the right shoulder, also owing to diaphragmatic irritation. The iliopsoas muscle has clinically important relations to the kidneys, ureters, cecum, appendix, sigmoid colon, pancreas, lumbar lymph nodes, and nerves to the posterior abdominal wall. If any of these structures is diseased, a positive psoas sign will be observed. The obturator sign may cause painful spasms of the adductor muscles of the thigh and sensory defi cits in the medial thigh.

142
Q

A 43-year-old man is admitted to the hospital with a knife wound to the right lobe of the liver. After a laparotomy is performed, digital pressure is applied to the hepatoduodenal ligament, but brisk bleeding continues, indicating a variation in the origin of the right hepatic artery. Which of the following is the most common variation in the arterial supply to the right lobe of the liver? ⃣ A. The right hepatic originates from the gastroduodenal. ⃣ B. The right hepatic originates from the superior mesenteric. ⃣ C. The right hepatic originates from the left gastric. ⃣ D. The right hepatic originates from the left hepatic. ⃣ E. The right hepatic originates directly from the aorta.

A

B. The most common variation in hepatic artery supply to the right lobe of the liver is the right hepatic artery originating from the superior mesenteric artery, in approximately 18% of cases.

143
Q

A 38-year-old woman is admitted to the hospital with signs of cholecystitis and gallbladder stones. During cholangiography, the catheter is inserted with diffi culty into the gallbladder. Which of the following structures is most likely to interfere with the passage of the catheter into the cystic duct? ⃣ A. Cystic duct compression by a hepatic artery ⃣ B. Spiral valve (of Heister) ⃣ C. Tortuosity of the cystic duct ⃣ D. Adhesions from the hepatoduodenal ligament ⃣ E. Portal vein compression of the cystic duct

A

B. The gallbladder consists of a fundus, body, and neck. The fundus is the rounded, blind end that comes in contact with the transverse colon. The body is the major part and rests on the upper part of the duodenum and transverse colon. The neck is the narrowest part and gives rise to the cystic duct. This duct contains the spiral valve (of Heister), which is a redundant mucosal fold that maintains patency of the duct. This is not actually a valve and does not determine the direction of fl ow of bile. This could potentially be a point of constriction that could present diffi culty with insertion of a catheter. The cystic duct comes into contact only with the cystic artery and is not particularly tortuous. The hepatoduodenal ligament is the thickened free edge of the lesser omentum, and it conducts the portal triad (portal vein, hepatic artery, and bile duct) and encloses structures that pass through the porta hepatis. This ligament is unlikely to compress the cystic duct. Though the cystic duct is in close relation to the portal vein and the hepatic artery, the most likely cause of diffi culty would be potential constriction by the spiral valve.

144
Q

A 57-year-old woman is admitted to the hospital with symptoms and signs of acute cholecystitis. Radiographic and physical examinations confi rm the initial diagnosis, and a cholecystectomy is performed. On the fi fth postoperative day the patient develops bile peritonitis. Which of the following conditions would most likely be responsible for such an outcome, assuming the cholecystectomy had been performed correctly? ⃣ A. The common bile duct is leaking. ⃣ B. The ducts of Luschka are leaking. ⃣ C. The right hepatic duct is leaking. ⃣ D. The cystic duct is leaking. ⃣ E. The left hepatic duct is leaking

A

B. The ducts of Luschka are accessory biliary ducts that are not present in all individuals. During a cholecystectomy the cystic duct and cystic artery are ligated and the gallbladder is removed, using sharp dissection to separate the gallbladder from the liver. Routinely, the right hepatic duct and left hepatic duct are not encountered directly. If the surgeon was unaware that ducts of Luschka were present in the patient, they would not have been ligated or clipped and their leakage would result in bile peritonitis. The common bile duct and hepatic ducts are left intact during the surgery and should not leak. The cystic duct is ligated during the surgery and with proper ligation would not produce bile peritonitis.

145
Q

A 64-year-old man is admitted to the hospital with intense abdominal pain from a pancreatic tumor. A neurectomy is performed to interrupt the neural fi - bers supplying the pancreas. Which of the following neural fi bers would be the most likely objective of the neurectomy? ⃣ A. Sympathetic preganglionic ⃣ B. Sympathetic postganglionic ⃣ C. Visceral afferent ⃣ D. Postganglionic preganglionic ⃣ E. Postganglionic postganglionic

A

C. Visceral afferents are nerve fi bers that transmit the sensation of visceral pain. The target of the neurectomy would be to eradicate visceral pain. One would not want to interfere with sympathetic and parasympathetic nerves, as these provide motor innervation to viscera.

146
Q

A 54-year-old man is admitted to the emergency department with a 2-day history of mild abdominal pain, in addition to bloating, nausea, vomiting, and poor appetite. Past medical history reveals that the patient has just recovered from a pneumonia infection. Radiographic examination reveals a paralytic ileus. Which of the followings signs would most likely be found during a physical examination? ⃣ A. Increased bowel sounds ⃣ B. Absent bowel sounds ⃣ C. Borborygmi ⃣ D. Crampy abdominal pain ⃣ E. Localized tenderness

A

B. Adynamic ileus is essentially paralysis of the bowel. It can result from many causes, including kidney stone, spinal injury, peritonitis, etc. Typically, bowel obstruction is characterized initially by increased borborygmi (bowel sounds). Mechanical obstruction can be caused by blockage within the bowel or compression of the bowel from an external source. Increased borborygmi usually follow such obstructions immediately. As the bowel muscle tires, however, bowel sounds can become reduced or absent. Although this patient might have peritonitis, with an abdomen tender to palpation, the data simply indicate generalized abdominal pain. Crampy pain has not been noted.

147
Q

A 65-year-old man is admitted to the emergency department with complaints of nonspecifi c abdominal pain. Physical and radiographic examinations reveal mild intestinal ischemia due to atherosclerotic occlusion of the midproximal part of the superior mesenteric artery, but collateral blood supply has delayed the onset of necrosis. What vessels provide collateral channels between the celiac trunk and the superior mesenteric artery? ⃣ A. Superior and inferior pancreaticoduodenal ⃣ B. Left gastric and hepatic ⃣ C. Cystic and gastroduodenal ⃣ D. Right and left colic ⃣ E. Right and left gastroomental

A

A. Blood supply from the inferior pancreaticoduodenal artery via the superior mesenteric artery can provide collateral blood supply to the head of the pancreas and the fi rst part of the duodenum in situations when the celiac trunk is occluded. Such anastomoses occur between the superior pancreaticoduodenal branches of the gastroduodenal artery (a derivative of the common hepatic branch of the celiac trunk) and the inferior pancreaticoduodenal. The left gastric artery and hepatic artery are derivatives of the celiac trunk and do not anastomose with the superior mesenteric. The cystic artery and the gastroduodenal artery are derivatives of the common hepatic from the celiac trunk. They would not typically provide an anastomosis between the celiac trunk and superior mesenteric artery (unless there is an aberrant right hepatic branch from the superior mesenteric artery). The right colic artery and left colic artery anastomose via the marginal artery of the colon, but this provides collateral supply of the inferior mesenteric artery. The right and left gastroomental arteries anastomose and provide collateral supply to the greater curvature of the stomach but are derived from the celiac trunk and thus do not provide communication between the celiac trunk and superior mesenteric artery.

148
Q

A 22-year-old man is admitted to the emergency department with acute abdominal pain at his right lower quadrant. Radiographic and physical examinations provide evidence of acute appendicitis. An appendectomy is performed, beginning with an incision at McBurney’s point. Through which of the following abdominal layers must the surgeon pass to reach the appendix through this incision? ⃣ A. External abdominal oblique muscle, internal oblique muscle, transversalis fascia, parietal peritoneum ⃣ B. Aponeurosis of the external abdominal oblique muscle, internal oblique muscle, transversus abdominis muscle, transversalis fascia, parietal peritoneum ⃣ C. Aponeurosis of the external abdominal oblique muscle, internal oblique muscle, transversus abdominis muscle, parietal peritoneum ⃣ D. Aponeurosis of the external abdominal oblique muscle, aponeurosis of internal oblique muscle, transversus abdominis muscle, transversalis fascia, parietal peritoneum ⃣ E. Aponeurosis of the external abdominal oblique muscle, aponeurosis of internal oblique muscle, aponeurosis of transversus abdominis muscle, transversalis fascia, parietal peritoneum

A

B. The incision and tissue separation at McBurney’s point to reach the appendix will usually encounter the aponeurosis of the external abdominal oblique muscle, internal oblique muscle, transversus abdominis muscle, transversalis fascia, and peritoneum. Muscle fi bers of the internal oblique and transversus can be separated bluntly, without cutting them. The appendix is located intraperitoneally within the abdomen, thus it is covered with visceral peritoneum. The transversalis fascia separates the internal surface of the transversus abdominis from the parietal peritoneum. Thus, fi ve layers must be penetrated to access the infl amed appendix.

149
Q

A 12-year-old male is admitted to the hospital with profuse rectal bleeding but appears to be free of any associated pain. Which of the following is the most common cause of severe rectal bleeding in the pediatric age group? ⃣ A. Internal hemorrhoids ⃣ B. External hemorrhoids ⃣ C. Diverticulosis ⃣ D. Ileal (Meckel) diverticulum ⃣ E. Borborygmi

A

D. Ileal (Meckel) diverticulum, which is an outpouching of the distal ileum, is twice as prevalent in males as in females. The diverticulum is clinically important because ulceration of the diverticulum with pain, bleeding, perforation, and obstruction is a complication that may require emergent surgery. Signs and symptoms frequently mimic appendicitis or peptic ulcer. Internal hemorrhoids are thrombosed tributaries of the middle rectal vein, which can prolapse into the anal canal. External hemorrhoids are thromboses in the veins of the external rectal venous plexus. Diverticulosis is ordinarily an outpouching of the wall of the large intestine. This primarily affects the aged and does not cause bleeding in most cases. Borborygmi are sounds created by gas and intestinal contents as they pass through the gastrointestinal tract.

150
Q

A 48-year-old male is admitted to the hospital with abdominal distension and pain. The radiographic image is shown in Fig. 3-2 . In which of the following locations will blood be detected with an ultrasound machine if the patient stands upright? ⃣ A. Subphrenic space ⃣ B. Hepatorenal space (pouch of Morison) ⃣ C. Rectouterine space (pouch of Douglas) ⃣ D. Rectovesical space ⃣ E. Subhepatic space

A

D. Fig. 3-2 is a CT scan that reveals an aortic aneurysm with hemorrhage. If the man stands in an erect position, the blood will be detected in the rectovesical space, which is a peritoneal recess between the bladder and the rectum in males. The subphrenic space is a peritoneal pouch between the diaphragm and the anterior and superior part of the liver. The hepatorenal space (pouch of Morison) is a deep peritoneal pocket between the posterior surface of the liver and the right kidney and suprarenal gland. The rectouterine space (pouch of Douglas) is a sac in females formed by a peritoneal-lined recess between the rectum and uterus. Although this is a logical answer choice, the rectouterine space is found only in women. The subhepatic space is between the liver and the transverse colon.

151
Q

A 27-year-old woman is admitted to the emergency department with markedly elevated temperature and abdominal pain. Physical examination initially indicates paraumbilical pain, but the site of origin of pain soon shifts to the right lower quadrant. A CT scan is shown in Fig. 3-3 . Which of the following structures is affected? ⃣ A. Right ovary ⃣ B. Appendix ⃣ C. Iliocecal junction ⃣ D. Ascending colon ⃣ E. Ileum

A

B. Paraumbilical pain progressing into the right iliac fossa is a sign of appendicitis. The CT scan is one of an infl amed appendix. The structures in the scan that lie to the right of the vertebral body are part of the psoas muscle. Transverse section of this muscle signifi es that it is a cut from the lumbar region.

152
Q

A 51-year-old complains of abdominal pain of 2-month duration. A CT scan of the patient’s abdomen is shown in Fig. 3-4 . An angiogram indicates that several arteries of the gastrointestinal tract are occluded due to atherosclerosis, producing bowel ischemia. Which of the following arteries is most likely occluded in the CT scan? ⃣ A. Middle colic ⃣ B. Right colic ⃣ C. Left colic ⃣ D. Iliocolic ⃣ E. Marginal

A

C. The arrows in the CT scan point to the descending colon. Therefore, the left colic artery, which supplies the descending colon, is the one most likely occluded in the CT scan. The middle colic, right colic, and ileocolic artery supply the ascending colon, and the marginal artery provides an anastomosis between branches of the superior mesenteric artery and inferior mesenteric artery.

153
Q

A 53-year-old man visits the outpatient clinic because of an abnormal mass developing in his anal canal. An image from the physical examination is seen in Fig. 3-5 . A biopsy of the tissue reveals squamous cell carcinoma of the anus. Which of the following lymphatics will most typically fi rst receive cancerous cells from the anal tumor? ⃣ A. Deep inguinal lymph nodes ⃣ B. Superfi cial inguinal lymph nodes ⃣ C. Internal iliac nodes ⃣ D. External iliac nodes ⃣ E. Paraaortic nodes

A

The superfi cial inguinal lymph nodes are the fi rst drainage site for superfi cial perineal structures like the skin of the perianal region, prepuce of penis, scrotum, and anal canal inferior to the pectinate line. The superfi cial inguinal nodes also drain structures of the inferolateral quadrant of the trunk, including the anterior abdominal wall inferior to the umbilicus, gluteal region, and the lower limb. The deep inguinal lymph nodes drain the glans of the penis and the distal spongy urethra and receive drainage from the superfi cial inguinal lymph nodes. The common iliac nodes drain the external and internal iliac lymph nodes. The internal iliac lymph nodes drain inferior pelvic structures and deep perineal structures and receive drainage from the sacral nodes. The paraaortic nodes are the fi nal drainage site for all the above lymph nodes before they drain to the thoracic duct by way of the lumbar lymph trunks.

154
Q

A 49-year-old man comes to the outpatient clinic with complaints of bloating, gas, and a sense of fullness for the preceding 2 years. A CT scan examination is shown in Fig. 3-6 . Which of the following structures is affected? ⃣ A. Spleen ⃣ B. Stomach ⃣ C. Duodenum ⃣ D. Pancreas ⃣ E. Descending colon

A

B. The white arrows in the CT scan in Fig. 3-6 point to the stomach. The spleen would not be in this CT slice because it is located more superiorly in the body than this section. Also, note that the psoas muscles are seen lying on the sides of the vertebral body, meaning that the section is in the lumbar region of the body. The duodenum is the structure located to the right of the stomach in the scan and is posterolateral to the aorta.

155
Q

An endoscopic examination is performed on a 49-year-old patient with a preliminary diagnosis of a gastrointestinal cancer, and a tissue sample is sent for histopathologic examination. Biopsy reveals a gastric adenocarcinoma, and a total gastrectomy is performed. Which of the following lymph nodes will most likely fi rst receive metastatic cells? ⃣ A. Celiac ⃣ B. Splenic ⃣ C. Suprapancreatic ⃣ D. Right gastric ⃣ E. Cisterna chyli

A

A. Cancerous cells in the stomach would metastasize fi rst to the celiac nodes. The splenic nodes are located along the splenic artery and are related to drainage from the pancreas; therefore, they will not be the fi rst to receive metastatic cells from a stomach cancer. The suprapancreatic nodes would be associated with pancreatic carcinoma. The right gastric lymph nodes may receive metastatic cells but would not be the fi rst to receive this lymph because they are located along the lesser curvature of the stomach. The cisterna chyli, the proximal expanded portion of the thoracic duct, receives lymph drainage from the entire abdomen; therefore, it would not be fi rst to receive cancerous cells.

156
Q

A 28-year-old female visits the outpatient clinic to receive the required physical examination for an insurance policy. Physical and laboratory examinations give evidence that she is probably a normal, healthy woman. A radiograph of the patient is shown in Fig. 3-7 . Which of the following is the most likely diagnosis? ⃣ A. Cholecystitis ⃣ B. Carcinoma of the liver ⃣ C. A caudal extension of the right hepatic lobe (Riedel lobe) ⃣ D. Pancreatic carcinoma ⃣ E. Carcinoma of the stomach

A

C. Because physical and laboratory tests show a normal, healthy woman, the anomaly of the radiograph would be expected to be benign. Riedel’s lobe is a normal variation of the liver, often an inferior extension of the right lobe of the liver, lateral to the gallbladder that extends about 4 or 5 cm below the rib cage. Carcinomas would present with abnormal laboratory examinations, and cholecystitis would present with an abnormal physical examination, as when the gallbladder is infl amed

157
Q

A 35-year-old woman is admitted to the hospital with a mass protruding through her skin at the right lower quadrant. Physical examination reveals intestinal herniation, as shown in Fig. 3-8 . Which of the following is the most likely diagnosis? ⃣ A. Richter hernia ⃣ B. Spigelian hernia ⃣ C. Paraumbilical ⃣ D. Incisional hernia ⃣ E. Ventral hernia

A

B. A Spigelian hernia occurs along the semilunar line below the umbilical region and can protrude through the skin. A Richter hernia is a hernia that presents as a strangulated segment of part of the wall of an intestinal loop through any hernial opening. A paraumbilical hernia occurs at the level of the umbilicus, near the midline. An incisional hernia occurs with dehiscence (breakdown and reopening) of an operative incision after surgery. A ventral hernia is a type of incisional hernia located on the ventral surface of the abdomen, occurring only after surgery.

158
Q

A 3-year-old male is admitted to the pediatric clinic with a palpable mass in the right side of his scrotum, and a preliminary diagnosis is made of a congenital, indirect inguinal hernia. Which of the following is the most likely cause of an indirect inguinal hernia in this patient? ⃣ A. The deep ring opens into an intact processus vaginalis. ⃣ B. Congenital hydrocele ⃣ C. Ectopic testis ⃣ D. Epispadias ⃣ E. Rupture of the transversalis fascia

A

A. Congenital inguinal hernias occur when a large patency of the processus vaginalis remains so that a loop of intestine herniates into the inguinal canal. A congenital hydrocele is also caused by a patent segment of a processus vaginalis fi lled with fl uid, but it does not cause an indirect hernia. Ectopic testes occur when the gubernaculum does not migrate correctly during development and the testis does not reach the scrotum, but this does not cause a hernia. Epispadias occurs when the external urethral orifi ce opens onto the dorsal surface of the penis and is generally associated with exstrophy of the bladder. A rupture, or tear, of the transversalis fascia would not cause the intestines to herniate through the deep inguinal ring and therefore would not cause an indirect inguinal hernia.

159
Q

A 43-year-old woman visits the outpatient clinic with complaints of chronic dysphagia and gastroesophageal refl ux. An endoscopic examination reveals achalasia of the cardia of the stomach. Which of the following is the most likely cause of this condition? ⃣ A. Failure of relaxation of the lower esophageal sphincter ⃣ B. Dyspepsia ⃣ C. Gastritis ⃣ D. Gastroparesis ⃣ E. Peptic ulcer

A

A. Failure of relaxation of the lower esophageal sphincter (also known as the cardiac sphincter) causes an accumulation of food in the esophagus. Achalasia is the failure of motility of food through the esophagus into the stomach. A constricted lower esophageal sphincter is the cause of these conditions. Dyspepsia is chronic pain or discomfort in the upper abdomen. This usually accompanies problems with digestion and is not associated with diffi culty swallowing. Gastritis is infl ammation of the mucosal lining of the stomach and would also not contribute to dysphagia. Gastroparesis is defi ned as delayed stomach emptying due to stomach paralysis, which would reveal chyme overloading in the stomach and esophagus (achalasia involves only the esophagus). Peptic ulcers mostly result in pain in the stomach, more commonly the duodenum, due to erosion of the mucosal lining.

160
Q

A 21-year-old male is admitted to the hospital with periumbilical pain. A CT scan examination reveals internal bleeding about 2 feet proximal to the ileocecal junction. Which of the following is the most likely diagnosis? ⃣ A. Ruptured appendix ⃣ B. Volvulus ⃣ C. Diverticulosis ⃣ D. Ileal (Meckel) diverticulum ⃣ E. Borborygmi

A

D. Meckel diverticulum is an embryologic remnant of the vitelline duct in the embryo located on the distal ileum and proximal to the cecum. If this diverticulum becomes infected, it produces pain in the umbilical region of the abdomen, in addition to possible bleeding. A ruptured appendix usually presents with pain in the lower right quadrant of the abdomen, when the infective processes come in contact with adjacent parietal peritoneum. A volvulus is characterized by a twisted bowel, which causes obstruction of the bolus and/or ischemia as the blood supply is occluded. Diverticulosis is a condition that causes outpouchings of the wall of the gut tube, usually found in the sigmoid colon. Pain from this condition would usually present in the lower left quadrant. Borborygmi are sounds produced from gas and other contents moving through the bowels. This would not cause pain in one specifi c area because peristaltic activity moves the length of the GI tract.

161
Q

A 45-year-old man is admitted to the hospital with pain in the right upper quadrant, with radiation to the tip of his scapula. Radiographic examination reveals gallbladder stones, with associated cholecystitis. An open cholecystectomy is performed, using a Kocher incision (along the right costal margin). Which of the following nerves are most likely at risk during this incision? ⃣ A. T5, T6 ⃣ B. T6 to T8 ⃣ C. T7, T8 ⃣ D. T9 to L1 ⃣ E. T5 to T9

A

C. Pain from the gallbladder is sent to the spinal cord by visceral afferents and also is mediated (referred) by nerve fi bers that provide pain sensation to the scapula. Open cholecystectomy would cause a risk to the T7 and T8 spinal nerves due to their close proximity to the gallbladder. These nerves are located below the associated rib and along the same horizontal plane as the gallbladder. T5 and T6 nerves are located superior to an incision and thus are not affected. For the same reason, nerves from T6 to T8 would not be the right choice due to T6 not being at risk during this procedure. Nerves from T9 to L1 are located inferior to the incision during this procedure. T5 to T9 is a broad range answer that includes many nerves that would not be affected by the incision.

162
Q

A 3-year-old female is admitted to the pediatric clinic because of a palpable right inguinal mass. An open surgical procedure is performed. Digital pressure is used to return organ contents of the hernia to the abdomen. A sac of peritoneum can be seen clearly, protruding from the internal ring. Which of the following terms is most accurate for the origin of this structure? ⃣ A. A patent processus vaginalis (canal of Nuck) ⃣ B. Congenital hydrocele ⃣ C. Ectopic uterus ⃣ D. Femoral hernia ⃣ E. Rupture of the transversalis fascia

A

A. The processus vaginalis is formed as the parietal peritoneum layer of the abdominal wall (inguinal region) evaginates through the deep inguinal ring and continues through the superfi cial inguinal ring. Normally, this evagination or outpouching is obliterated during development. A cyst can develop in a segment of the processus (which is also referred to as the canal of Nuck) if this processus is not obliterated. Congenital hydrocele would present at the base of the canal; in this case, the swelling would be in the labium majus. An ectopic uterus would present as a mass in the pelvis and not the inguinal region. A femoral hernia would be palpated below the inguinal ligament (usually) just medial to the femoral triangle. A defect of the transversalis fascia could result in infl ammation in a specifi c area but would not be located along the inguinal ligament because this fascial layer is located deep to the inguinal ligament.

163
Q

A 48-year-old female was scheduled for radiographic examination because of severe abdominal pain. The imaging gave evidence of a pancreatic pseudocyst. Which of the following is the typical topographic location for this type of pseudocyst? ⃣ A. Right subhepatic space ⃣ B. Hepatorenal space ⃣ C. Omental bursa ⃣ D. Right subphrenic space ⃣ E. Greater sac

A

C. The most likely place that a pancreatic pseudocyst will be formed is in the fl oor of the omental bursa, deep to the stomach. The omental bursa is a potential space behind the stomach and directly anterior to the pancreas. Pancreatic extravasations will fi ll this space. The right subhepatic space is the space in the peritoneal cavity between the inferior visceral surface of the liver and the transverse colon. The hepatorenal space of the subhepatic space, also known as the pouch of Morison, is located between the right lobe of the liver and the parietal peritoneum covering the superior pole of the right kidney and suprarenal gland. The right subphrenic space is the space directly inferior to the diaphragm and above the diaphragmatic surface of the liver. It is above the pancreas; therefore, fl uid from the pancreas could not accumulate there. Finally, the greater sac is the general peritoneal cavity of the abdomen. The greater sac communicates with the omental bursa (lesser sac) by way of the omental (epiploic) foramen (of Winslow). The peritoneal cavity contains nothing except a very thin fi lm of serous fl uid that allows the organs to slip around relatively freely against one another and on the body wall.

164
Q

A 38-year-old male is examined in the outpatient clinic because of his complaint of mild abdominal pain of 2 years’ duration. Upon examination, it is observed that the pain is dull and located principally in the left upper quadrant around the xiphoid process. An endoscopic examination reveals that the patient suffers from a gastric ulcer. At which of the following spinal nerve levels are the neuronal cell bodies located for the sensory fi bers in such a case of gastric ulcer? ⃣ A. T5, T6 ⃣ B. T6 to T8 ⃣ C. T7, T8 ⃣ D. T9 to L1 ⃣ E. T5 to T9

A

C. The spinal cord levels containing the soma of the sensory fi bers transporting the sensation of pain are more than likely at the level of T7 and T8. This is because the xiphoid process is at these dermatome levels for somatic sensations of pain, and these same spinal nerves receive visceral afferents from the stomach.

165
Q

A 43-year-old man is admitted to the hospital with abdominal pain and vomiting. A CT scan examination reveals that the patient has an internal hernia involving his duodenum. An exploratory laparotomy reveals a paraduodenal hernia. Which of the following arteries is most at risk during the repair of this hernia? ⃣ A. Middle colic ⃣ B. Sigmoidal ⃣ C. Ileocolic ⃣ D. Ileal ⃣ E. Ascending branches of left colic

A

E. The ascending branches of the left colic artery are at risk during repair of a paraduodenal hernia because the location of this hernia is in the upper left quadrant, adjacent to the junction of the terminal duodenum and the jejunum. The ascending branches of the left colic artery supply the upper segment of the descending colon and the splenic fl exure of the transverse colon. The middle colic artery arises from the superior mesenteric artery and supplies the ascending colon and the transverse colon and anastomoses with the left colic artery. The right colic artery is a more inferior branch of the superior mesenteric artery and supplies the proximal ascending colon. The ileocolic artery supplies the ileum and large intestine in the area of the ileocecal junction. Finally, the ileal arteries are the small terminal branches of the superior mesenteric artery supplying blood to the ileum.

166
Q

A 42-year-old male patient with jaundice is admitted to the hospital with severe pain that radiates to his back. A CT scan examination reveals a tumor at the neck of the pancreas. Biopsy reveals a ductular adenocarcinoma. Which of the following structures will fi rst receive metastatic cells? ⃣ A. Stomach ⃣ B. Spleen ⃣ C. Duodenum ⃣ D. Liver ⃣ E. Vertebral column

A

D. The liver would be the fi rst structure to receive these metastatic cells because they would fl ow through the portal venous system from the pancreas to the liver. The stomach would not receive these cells because there is no communication between the stomach and pancreas through circulatory or ductal pathways. The spleen also does not have direct communication with the pancreas and would not receive metastases fi rst. The duodenum is the site for pancreatic emptying, but as these metastases pass through venous circulation they would not pass into the duodenum. The vertebral column would not receive the metastases because they would not enter the vertebral venous plexus.

167
Q

A major vessel appears to be nearly occluded in a 42-year-old male patient diagnosed with ductular adenocarcinoma. A CT scan examination has clearly demonstrated the tumor is at the neck of the pancreas. Which of the following vessels would be the most likely to be obstructed? ⃣ A. Inferior mesenteric vein ⃣ B. Portal vein ⃣ C. Superior mesenteric artery ⃣ D. Posterior superior pancreaticoduodenal artery ⃣ E. Greater pancreatic artery

A

B. The portal vein is the most likely structure to be occluded by a large tumor at the neck of the pancreas. The pancreas is drained via the splenic vein and empties into the portal vein. It directly enters the liver through the portal vein. The superior mesenteric vein drains the small intestines and the proximal part of the large intestine. The posterior superior pancreaticoduodenal artery would never receive these metastases, nor would the greater pancreatic artery.

168
Q

A 49-year-old woman is admitted to the hospital with abdominal pain. Physical examination reveals an epigastric pain that migrates toward the patient’s right side and posteriorly toward the scapula. Radiographic examination reveals cholecystitis with a large gallstone and no jaundice. In which of the following structures is it most likely the gallstone will be located? ⃣ A. Common bile duct ⃣ B. Hartmann pouch ⃣ C. Left hepatic duct ⃣ D. Pancreatic duct ⃣ E. Right hepatic duct

A

B. The Hartmann’s pouch is located in the gallbladder at the junction of the neck and the cystic duct. When a gallstone is located in this area, the patient will present with pain but usually no jaundice due to no occlusion of the cystic duct. A common bile duct and/or left and right hepatic duct obstruction would cause posthepatic jaundice due to bile obstructed in the duct system. Obstruction of the pancreatic duct would cause pain in the umbilical region, not in the right upper quadrant.

169
Q

A 47-year-old female is admitted to the hospital with jaundice and epigastric pain that migrates toward the patient’s right side and posteriorly toward the scapula. Radiographic examination reveals cholecystitis with a large gallstone. Which of the following is the most likely site for a gallstone to lodge? ⃣ A. Common bile duct ⃣ B. Hepatopancreatic ampulla ⃣ C. Left hepatic duct ⃣ D. Pancreatic duct ⃣ E. Right hepatic duct

A

B. The hepatopancreatic ampulla is also known as the ampulla of Vater and is located at the junction of the pancreatic duct and common bile duct. It is the narrowest part of the ductal system. The common bile duct, left hepatic duct, pancreatic duct, and right hepatic duct all have larger diameters than the ampulla of Vater.

170
Q

A 40-year-old male with nine children is urged by his wife to have a vasectomy. During the operation the urologist separates the layers of the spermatic cord to expose the ductus deferens so that it can be ligated and cut. From what structure is the internal spermatic fascial covering derived? ⃣ A. Internal oblique muscle ⃣ B. Cremaster muscle ⃣ C. External abdominal oblique muscle fascia ⃣ D. Transversus abdominis aponeurosis ⃣ E. Transversalis fascia

A

E. The innermost covering of the spermatic cord is the internal spermatic fascia. It originates from the transversalis fascia. The internal oblique muscle provides origin for the cremaster layer of the cord. The external spermatic fascia is continuous from the external oblique aponeurosis and its fascia. The transversus abdominis aponeurosis plays no part in the formation of the spermatic cord.

171
Q

A 26-year-old female patient has a nonpainful hernia in the midline of the abdominal wall, several inches above the level of the umbilicus. Which of the following hernias will most likely characterize this condition? ⃣ A. Umbilical hernia ⃣ B. Spigelian hernia ⃣ C. Epigastric hernia ⃣ D. Femoral hernia ⃣ E. Omphalocele

A

C. An epigastric hernia is formed by a weakness in the intersecting fi bers of the linea alba superior to the umbilicus. In most cases, herniation of fat and other tissue through the defect causes a palpable, but painless, mass. If a nerve branch also passes through the defect, it can be associated with local pain. Umbilical hernias are common in newborn babies and pregnant females in the third trimester of pregnancy. This kind of hernia usually represents a weakness in the wall structure at the level of the umbilicus. Omphaloceles are a more serious (but less common) defect, representing failure of intestines to return to the abdominal cavity, associated with lack of proper growth of the body wall. Spigelian hernias occur through the semilunar lines, lateral to the rectus sheath. Femoral hernias pass through the femoral canal, deep to the inguinal ligament.

172
Q

A 23-year-old male suffered a knife wound to the epigastric region. At laparotomy, when the abdomen is opened for inspection, it is seen that an injury to the liver occurred between the bed of the gallbladder and the falciform ligament, and the wound is bleeding profusely. The Pringle maneuver is performed with a nontraumatic vascular clamp, but blood continues spurting from the surface of the liver. Which part of the liver and which artery is most likely injured? ⃣ A. Lateral segment of the left lobe and the left hepatic artery ⃣ B. Caudate segment of the liver, with injury both to right and left hepatic arteries ⃣ C. Anterior segment of the right lobe, with injury to the right hepatic artery ⃣ D. Medial segment of the left lobe, with injury to an aberrant left hepatic artery ⃣ E. Quadrate lobe, with injury to the middle hepatic branch of the right hepatic artery

A

D. The knife injured the medial segment of the left lobe of the liver, located between the falciform ligament and the gallbladder. This area of the liver is usually supplied by the left hepatic artery. If this were the case, the Pringle maneuver (compression of the hepatoduodenal ligament) would slow or stop the bleeding. An aberrant, left hepatic branch of the left gastric artery does not pass through the hepatoduodenal ligament, however, and is therefore not compressed in a Pringle maneuver; thus, the bleeding is not reduced by that technique. The lateral segment of the left lobe is located to the left of the falciform ligament. The caudate segment of the liver is located in the inferior aspect of the upper portion of the liver, well above the site of injury. The anterior segment of the right lobe is located to the right of the gallbladder. The right lobe receives its arterial supply from the right hepatic artery.

173
Q

A 47-year-old male patient had undergone bilateral vagotomy, with division of both vagus trunks at the esophageal hiatus, to relieve his chronic diffi culty with peptic ulcers. Which of the following conditions will most likely occur? ⃣ A. Parasympathetic supply to the descending colon is lost. ⃣ B. The patient would no longer have contraction of the urinary bladder. ⃣ C. The patient would become impotent. ⃣ D. The patient would be sterile because of paralysis of the ductus deferens and ejaculatory duct. ⃣ E. Parasympathetic supply to the ascending colon would be reduced or absent.

A

E. Interruption of both vagus nerves would deprive the abdominal viscera of parasympathetic supply, that is, to the level of the splenic fl exure of the colon. Distal to the splenic fl exure, the colon receives parasympathetic nerves. Pelvic splanchnic nerves supply the descending colon, the urinary bladder, and the erectile tissues of the penis. The innervation of the ductus deferens and ejaculatory duct is carried by sympathetic nerve supply through the pelvic plexuses. Parasympathetic supply to the ascending colon is carried by the vagus nerves and would be lost.

174
Q

A 35-year-old male, an accountant, was brought to the emergency department with the complaint of intense abdominal pain of one-hour duration. His abdomen was distended, rigid, did not move in respiration, and was painful to palpation. He had adynamic (paralytic) ileus resulting from a peptic ulcer, although there was very little bleeding into the peritoneal cavity. He complained that he had pain in his right shoulder. Which of the following conditions will most likely occur? ⃣ A. Radiographs would not reveal the presence of air under his diaphragm. ⃣ B. Borborygmi would be decreased in frequency and amplitude. ⃣ C. He probably suffered from a posterior penetrating ulcer rather than from an anterior perforating ulcer. ⃣ D. The patient’s ulcer probably occurred in the second part of the duodenum. ⃣ E. The patient probably had acute appendicitis.

A

B. Adynamic ileus is paralysis of the bowel. Peristaltic activity ceases. Borborygmi (bowel sounds) are absent when this occurs, as in peritonitis. Radio- graphs would indicate the presence of air under the diaphragm. Pain in the shoulder is due to air under the diaphragm from the perforated anterior duodenal wall; this air irritates visceral afferent pain fi bers of the diaphragm, carried by the right phrenic nerve to spinal nerve levels C3 to C5. Referral of pain to the shoulder occurs because somatic sensory fi bers from the shoulder enter the spinal cord at similar levels. A posterior penetrating ulcer would be associated usually with profuse bleeding, mostly from branches of the gastroduodenal artery that supply the duodenal bulb, the fi rst part of the duodenum. Acute appendicitis is not associated with shoulder pain and adynamic ileus. A perforated appendix, however, will produce symptoms of peritonitis.

175
Q

A 68-year-old woman had been suffering longterm effects of diverticulosis and infl ammation of the transverse colon. To permit operating on a patient with severe diverticulosis of the transverse colon, it would be necessary to fi rst ligate (tie off) or clamp the source of arterial supply. Which of the following arteries will most likely be ligated? ⃣ A. Middle colic ⃣ B. Right colic ⃣ C. Superior mesenteric ⃣ D. Ileocolic ⃣ E. Left colic

A

A. The middle colic artery is the principal source of arterial supply to the transverse colon. The right colic artery, an infrequent branch of the superior mesenteric artery, supplies the ascending colon. The ileocolic branch of the superior mesenteric artery supplies distal ileum, cecum, and ascending colon. The left colic artery provides blood supply to the descending colon.

176
Q

A hard mass (a fecolith) in the ostium of the 27-year-old patient’s appendix had led to a local infection (appendicitis) with a slightly elevated temperature and a moderate increase in WBC count. The initial pain from the infection was dull and diffi cult to localize, but the patient placed his hand in the periumbilical area to indicate the general area of discomfort. The region of the umbilicus receives its sensory supply, classically, from which of the following spinal nerves? ⃣ A. T7 ⃣ B. T8 ⃣ C. T10 ⃣ D. T12 ⃣ E. L1

A

C. The dermatome of spinal nerve level T10 crosses the level of the umbilicus; that of T7 is at the level of the xiphoid process. T8 and T9 dermatomes lie between the two preceding spinal nerve levels. T12 innervates the lowest portion of the rectus abdominis and overlying skin with motor and sensory supply, respectively. L1 distribution by iliohypogastric and ilioinguinal nerves supplies the suprapubic region, the pubic area, and anterior portions of the urogenital region. Pain from appendicitis is most often perceived at fi rst in the periumbilical region, refl ecting the level of embryologic spinal nerve supply to the appendix. When the appendix swells and/or ruptures and contacts the body wall, somatic sensory fi bers of the adjacent body wall cause the apparent site of pain to shift to the lower right abdominal quadrant.

177
Q

A 55-year-old male had been unsuccessfully treated for alcoholism for 3 years. He was admitted to the hospital for emergency medical treatment for severe portal hypertension. Which of the following is a feature of the development of severe portal hypertension? ⃣ A. Esophageal varices—from increased pressure in the right gastric vein ⃣ B. Ascites—from effusion of fl uid from the inferior mesenteric vein ⃣ C. Internal hemorrhoids—from increased pressure within the superior mesenteric vein and its tributaries ⃣ D. Expansion of veins within the falciform ligament, which anastomose with veins of the umbilical region ⃣ E. Recanalization and expansion of the vessels within the medial umbilical ligaments

A

D. Caput medusae is an end-stage characteristic of liver cirrhosis. The snakelike appearance of veins on the body wall results from anastomoses between tiny veins that accompany the ligamentum teres (that is within the falciform ligament) with veins of the body wall. The umbilical veins are expanded, due to portal hypertension. Esophageal varices result from portal-systemic anastomoses between the left gastric vein and submucosal esophageal veins. Ascites is formed by fl uid transudate from thin-walled and dilated anastomotic vessels joining retroperitoneal intestinal veins and veins of the body wall. Internal hemorrhoids result from expansion of anastomoses between superior rectal tributaries to the inferior mesenteric vein and middle rectal branches of the internal iliac vein. Anastomoses between middle rectal veins and inferior rectal branches of the internal pudendal vein of the perineum result in external hemorrhoids.

178
Q

In performing a laparoscopic hernia repair on a 24-year-old female gymnast, the surgical resident observed the bright refl ection provided by the tissues of the iliopubic tract. The iliopubic tract could be traced medially to the site of the femoral herniation. The ilio- pubic tract is characterized by which of the following statements? ⃣ A. The iliopubic tract represents the aponeurotic origin of the transversus abdominis. ⃣ B. The iliopubic tract forms the lateral border of the inguinal triangle (of Hesselbach). ⃣ C. The iliopubic tract forms the lateral border of the femoral ring. ⃣ D. The iliopubic tract is the part of the inguinal ligament that attaches to the pectineal ligament. ⃣ E. The iliopubic tract is the lateral extension of the pectineal ligament.

A

A. The iliopubic tract is a refl ective band of aponeurotic tissue of the origin of the transversus abdominis, when visualized with the laparoscope. The lateral border of the inguinal triangle (of Hesselbach) is provided by the inferior epigastric artery and vein. The lateral border of the femoral ring is the femoral vein and connective tissue separating the vein from the femoral canal. The part of the inguinal ligament that attaches to the pectineal ligament is the lacunar ligament (of Gimbernant). The pectineal ligament becomes less dense and thinner as it is traced laterally from the femoral artery toward the iliopectineal portion of the inguinal ligament.

179
Q

47-year-old male had been scheduled for an appendectomy. During the open operative procedure it was discovered that atypical embryologic rotation of the intestine, adhesions, and adipose tissue made it somewhat diffi cult to fi nd the appendix. Most commonly, the vermiform appendix is best located by locating and tracing which of the following? ⃣ A. Anterior cecal artery ⃣ B. Descending branch of the right colic artery ⃣ C. Ileum to the ileocecal juncture ⃣ D. Posterior cecal artery ⃣ E. Taeniae coli of the ascending colon

A

E. Taeniae coli are a characteristic feature of the colon. Those of the cecum can be traced inferiorly to the base of the appendix, even when the appendix is retrocolic or retroileal in position, hidden thereby. The posterior cecal artery, although it provides origin to the appendicular artery, is very diffi cult to fi nd quickly, especially in the presence of malrotation and much adipose tissue. The other structures listed do not lead easily to the location of the appendix.